Nephrology Quiz and Questionnaire: 2009: Special Feature

You might also like

Download as pdf or txt
Download as pdf or txt
You are on page 1of 20

CJASN ePress. Published on May 24, 2010 as doi: 10.2215/CJN.

00540110
Special Feature

Nephrology Quiz and Questionnaire: 2009


Richard J. Glassock (Moderator)*; Participants: Joanne M. Bargman,† Biff F. Palmer,‡
Millie Samaniego,§ and Fernando C. Fervenza储
*Department of Medicine, Geffen School of Medicine at UCLA, Los Angeles, California; †Department of Medicine,
Toronto General Hospital, Toronto, Ontario, Canada; ‡Department of Internal Medicine, University of Texas
Southwestern Medical Center, Dallas, Texas; §Division of Nephrology, Department of Medicine, University of Michigan
Medical School, Ann Arbor, Michigan; and 储Division of Nephrology and Hypertension, Mayo Clinic, Rochester,
Minnesota
Clin J Am Soc Nephrol 5: 1141–1160, 2010. doi: 10.2215/CJN.00540110

T
he Nephrology Quiz and Questionnaire returns to the gressive renal impairment. In 2005, he underwent nephrectomy
pages of CJASN after an absence of 3 years, but it still for renal cell carcinoma. This resulted in worsening nephrotic
remains one of the most popular sessions at the annual syndrome and stage 5 chronic kidney disease. A peritoneal
meeting of the American Society of Nephrology. The meeting in dialysis (PD) catheter was inserted in July 2005, and the patient
2009 in San Diego was no exception, with a full-house atten- started on continuous ambulatory PD, 3 ⫻ 2 L exchanges
dance, estimated at more than 800 eager nephrologists. Eight during the day and icodextrin exchanges overnight. He devel-
challenging and educational cases were presented and dis- oped anuria over the subsequent 2 months. Peritonitis was
cussed by four able and skilled experts. The discussants were diagnosed on September 10, 2006, when he presented with
asked to prepare vignettes of puzzling cases, each illustrating abdominal pain and cloudy drainage fluid.
some topical, challenging, or controversial aspect of diagno- Examination of the PD drainage fluid revealed a white blood
sis or management of ESRD and dialysis (Dr. Bargman), fluid cell count of 1870 ⫻ 106/L, 90% neutrophils. He was started on
and electrolytes (Dr. Palmer), kidney transplantation (Dr. empiric therapy with intraperitoneal cefazolin and ceftazidime.
Samaniego), and glomerular disease (Dr. Fervenza). One or two Culture and sensitivity of the PD fluid revealed Acinetobacter
single best answers questions followed each case presentation, calcoaceticus-baumanii complex that was resistant to cefazolin.
which were addressed by the audience in a live electronic He was continued on intraperitoneal ceftazidime. Fungal pro-
response system. In addition, several weeks before the meeting, phylaxis was given with oral mycostatin.
the cases and questions (without the answers) were sent to all Subsequent PD fluid cell counts were as follows:
of the US nephrology training program directors as a question-
• September 12: ⬍100/L
naire. Their responses to the questions were tallied and pre-
• September 14: ⬍100/L
sented after the audience response but before the answers to the
• September 16: ⬍100/L
quiz and the analyses of the cases were given by the discus-
• September 20: 1360 ⫻ 106/L, 96% neutrophils
sants.
Each discussant prepared a manuscript summarizing his or
her discussion of the cases, an edited version of which serves as
Question 1
With respect to the increase in cell count on day 10 (September
the main text of this article. We hope that this “distillate” from
20) of therapy, which ONE of the following statements is MOST
San Diego will serve the subscribers to CJASN well and provide
correct (Figure 1)?
some fresh insights into the complexity and vibrancy of clinical
nephrology for those who were unable to attend (Richard J. A. It is unlikely to be fungal because of the prophylaxis with
Glassock, MD, Moderator). mycostatin.
B. The most likely cause is that the patient stopped taking his
Case 1: Joanne M. Bargman intraperitoneal antibiotic.
The patient was a 59-year-old man from North Africa. A diag- C. The causative organism became resistant to the antibiotic.
nosis of ankylosing spondylitis was made in 1980 and was D. The icodextrin has led to a chemical peritonitis via contam-
treated with anti-inflammatory agents. In 2002, he was noted to ination with peptidoglycans.
have significant proteinuria and microscopic hematuria. A re- E. The blood levels of ceftazidime became subtherapeutic.
nal biopsy demonstrated AA amyloidosis. He had slowly pro-
Discussion of Case 1 (Question 1)
The best answer is choice C: The causative organism became
Published online ahead of print. Publication date available at www.cjasn.org. resistant to the antibiotic. Acinetobacter calcoaceticus-baumanii
Correspondence: Dr. Richard J. Glassock, 8 Bethany, Laguna Niguel, CA 92677. belongs to the SPICE group of organisms. This is an acronym
Phone: 949-388-8885; Fax: 949-388-8882; E-mail: glassock@cox.net for a group of bacteria that have chromosomally mediated

Copyright © 2010 by the American Society of Nephrology ISSN: 1555-9041/506 –1141


1142 Clinical Journal of the American Society of Nephrology Clin J Am Soc Nephrol 5: 1141–1160, 2010

immunosuppressive drugs. Perhaps the greatest risk for yeast


or fungal peritonitis is the use of broad-spectrum antibiotics,
especially when instilled in the peritoneal cavity (6). It is pos-
tulated that the antibiotics disturb the usual balance between
bacteria and fungi within the bowel lumen, allowing for over-
growth of the fungi. Consequently, the extra burden of intralu-
minal fungi leads to migration of these organisms across the
bowel wall and into the peritoneal cavity. Although there are
anecdotal reports of eradication of these infections with anti-
fungal agents, this is a rare event, and, in most cases, the
catheter must be removed to treat the infection (7). After a
period off PD, usually ⱖ6 weeks, another catheter can be re-
inserted; however, fungal peritonitis is often associated with
the formation of intraperitoneal adhesions that may prevent the
successful resumption of PD. Unfortunately, it is difficult to
predict who will develop adhesions, and they cannot be im-
Figure 1. Answers from the membership, question 1.
aged, so it is impossible to know whether another catheter can
be inserted successfully until the time of the procedure. The use
of antifungal prophylaxis concurrent with broad-spectrum an-
activity that induces resistance to antibiotics (1). Although this tibiotics has been advocated to reduce the fungal overgrowth
group of organisms may be reported as sensitive to cephalo- and thus reduce the risk for a secondary fungal peritonitis.
sporins upon initial testing, they can quickly undergo genetic Results of studies using this approach have been variable, but
mutation with selection pressure for ␤-lactamase–producing in centers where there is a higher baseline rate of fungal peri-
mutants and become resistant to this class of antibiotic. Bacteria tonitis, the use of prophylaxis seems to reduce this complica-
included in the SPICE category include
tion (8,9). Given that a regimen such as oral mycostatin is
Serratia
associated with few complications, it may be worthwhile to
Pseudomonas/Providencia
co-administer oral mycostatin at the same time as the patient is
Indole-positive Proteus/Acinetobacter/Morganella
taking antibiotics, given the poor outcomes that are associated
Citrobacter
with fungal peritonitis. Although answer A addresses the re-
Enterobacter
duction in risk for fungal peritonitis because of the prophylaxis,
The initial response of the patient’s peritonitis to the cepha-
it is an overstatement to say that this complication is “unlikely”
losporin-based therapy speaks to the initial sensitivity of the
because of it. Certainly with a “relapse” during antibiotic ther-
Acinetobacter to these antibiotics; however, by the 10th day of
apy, fungal peritonitis must be considered a possibility. An-
treatment, the peritonitis relapsed because of the resistance that
swer B is not likely. First, the time to relapse in this case is
supervened. Fortunately, the organism remained sensitive to
particularly short for someone who stopped taking antibiotic.
aminoglycoside, his antibiotics were adjusted accordingly, and
Second, it is underappreciated how painful peritonitis is (10),
the peritonitis was cured. For serious infections caused by
SPICE or extended-spectrum ␤-lactamase organisms, the anti- and so a home dialysis patient, who already undergoes dialysis
biotics of choice are the carbapenems (2). autonomously, will be motivated to continue the antibiotic
The incidence of inducible resistance to antibiotics varies in therapy.
different parts of the world and these bacteria may also possess Icodextrin is a starch-based polymer that is used as an alter-
factors that confer resistance to other antibiotics such as the native to dextrose solutions for ultrafiltration in PD. The dex-
fluoroquinolones and aminoglycosides (3). It is important to be trins of dispersed molecular weight exert an oncotic pressure
aware of this activity in the SPICE group of bacteria. Otherwise, across the peritoneal capillary network, resulting in slow but
for example, the peritonitis in the patient discussed may have sustained ultrafiltration. In patients who are rapid transporters
been considered resistant to treatment and his PD catheter and have early dissipation of the glucose-based osmotic gradi-
removed. Several methods are available to detect extended- ent and loss of ultrafiltration over the long dialysis dwells,
spectrum ␤-lactamase activity, but they may be confounded by icodextrin has proved very helpful. Adverse effects are mini-
other factors that contribute to ␤-lactam resistance, such as mal, although skin rashes have been reported as a reaction to
production of Amp-C ␤-lactamase by Enterobacteriaceae (4). For this solution (11). Several years ago, several cases of culture-
this and many other reasons, it is helpful to have an infectious negative peritonitis were described with the use of icodextrin.
disease specialist with an interest in PD peritonitis work in This peritoneal reaction was found to be related to contamina-
conjunction with a PD program. tion of the solution by peptidoglycans that originated in the
Fungal peritonitis is a very serious complication in PD and is starch product, and modification in the production of this
associated with technique failure and death (5). Patients who solution has resulted in a reduction in this complication (12).
are especially at risk for this type of peritonitis include the That is why answer D is not the best choice. In addition, the
elderly, patients with diabetes, and those who are receiving chemical peritonitis that is associated with the peptidoglycans
Clin J Am Soc Nephrol 5: 1141–1160, 2010 Nephrology Quiz and Questionnaire: 2009 1143

typically manifests with a lower peritoneal cell count with less


neutrophilic predominance.
Finally, with instillation of high dosages of antibiotics into
the peritoneal cavity, there will be equilibration of these agents
with the blood compartment. Re-diffusion of the antibiotic into
the peritoneal cavity with subsequent PD exchanges will bring
more antibiotic into the peritoneal fluid; however, the principal
bacterial killing occurs with the very high dosages of antibiotic
that is usually given on a daily basis in the treatment of PD
peritonitis (13); therefore, answer E, which suggests that blood
concentration of antibiotic (rather than intraperitoneal concen-
tration) is important in treating the peritonitis, is not correct.

Case 2: Joanne M. Bargman


A 55-year-old right-handed woman with type 2 diabetes and
progressive diabetic nephropathy has been followed in a mul-
Figure 2. Answers from the membership, question 2.
tidisciplinary predialysis clinic for 2 years. She has received
education on options for renal replacement therapy. There are
no candidates to donate a kidney, and she declines home PD.
She undergoes left radiocephalic fistula creation, but the fistula E. Continue the intravenous cefazolin for 3 weeks, do not
fails to mature. She is assessed for fistula creation on the right change or re-site the tunneled internal jugular line, and
arm, but venous studies demonstrate in the right innominate institute an antibiotic lock.
vein a stenosis that is believed to be a sequela of a previous
central line during an intensive care unit admission for urosep- Discussion of Case 2 (Question 2)
sis. The correct answer is A: Continue the intravenous cefazolin for
While she is waiting to be reassessed by the vascular sur- 3 weeks total, and change the dialysis line over a guidewire.
geon, she has another episode of urosepsis and develops symp- The use of tunneled subclavian catheters and, more recently,
tomatic uremia and volume overload. She is admitted to hos- tunneled internal jugular catheters, has made it possible to have
pital, and a tunneled cuffed catheter is placed via the left immediate venous access for urgent or emergent HD and al-
internal jugular vein. She is started on hemodialysis (HD) three lows the patient to leave hospital and await a more permanent
times a week. vascular access. Unfortunately, the permanent (and superior)
After 1 month of HD, there is no evidence of return of renal form of vascular access, such as an arteriovenous fistula, often
function. She is reluctant to proceed with creation of another never is established. First, the patient may become comfortable
fistula. In the sixth week of therapy, she develops fever and with the internal jugular line, both because it avoids needle
chills on dialysis. There is no evidence of infection at the exit stick and because of a reluctance to undergo more procedures.
site and no hemodynamic compromise. There is no other ob- Second, the patient may agree to creation of an arteriovenous
vious source of infection. A presumptive diagnosis of line fistula but that surgery is not successful or the fistula is created
sepsis is made, and she is treated as an outpatient with intra- but does not develop sufficiently to support the HD blood
venous cefazolin and gentamicin. Blood cultures grow coagu- flows. According to recent data from the US Renal Data System,
lase-negative Staphylococcus that is sensitive to cephalosporins. a majority of patients start HD with “temporary” venous ac-
She is prescribed cefazolin at each HD session for 3 weeks. cess, and even many months later, half or more of these patients
Three days later at the HD unit, she has had no fever and does still have a catheter.
not develop fever or chills during the dialysis run. Catheter-related bacteremia (CRB) is a frequent complication
of indwelling HD catheters and is associated with a number of
serious complications, including septicemia, hospitalization,
Question 2
At this point, which ONE of the following is the BEST approach metastatic infection, and death. Indeed, the increased risk for
to her subsequent treatment (Figure 2)? mortality that results from an episode of CRB does not dissipate
with treatment of infection but instead is associated with an
A. Continue the intravenous cefazolin for 3 weeks total, and ongoing increased death risk for several years (14).
change the dialysis line over a guidewire. CRB that is associated with ongoing septic symptoms, in-
B. Discontinue the intravenous cefazolin after 1 week, and cluding fever and hypotension, is usually treated with antibi-
switch to oral therapy for 2 weeks. otics, systemic support, and removal of the dialysis catheter.
C. Continue the intravenous cefazolin for 3 weeks total, mon- Similarly, the catheter is removed when there is CRB in asso-
itor the exit site, but change the line only if there is evidence ciation with infection at the exit site. What is unclear is whether
of infection at the exit site. the catheter has to be removed when there is an “uncompli-
D. Continue the intravenous cefazolin for 3 weeks, and re-site cated” CRB—that is, unassociated with hemodynamic compro-
the tunneled venous catheter to the femoral vein. mise or exit-site infection.
1144 Clinical Journal of the American Society of Nephrology Clin J Am Soc Nephrol 5: 1141–1160, 2010

Unfortunately, most studies to date have suggested that an- a relatively newer procedure, however, I am reserving judg-
tibiotic therapy alone without removal or exchange of the HD ment and suggest that option A is the better one. Of course,
catheter is associated with an unacceptable rate of relapse of the from the vantage of the patient, an antibiotic lock would be
bacteremia. Only one third of catheters could be “salvaged” preferable to a line change.
with this approach, and, in the majority, the infection recurred
such that the catheter eventually had to be removed (15,16). Case 3: Biff F. Palmer
Moreover, there seems to be a higher incidence of metastatic A 33-year-old black woman is admitted with right flank pain
infection compared with those who undergo immediate re- radiating to the groin in association with gross hematuria. Her
moval of the infected line (17,18). A recent study from the history is noteworthy for one previous episode of nephrolithi-
United Kingdom reported a salvage rate in two thirds of bac- asis 6 months before. The patient works as a fashion model and
teremias, but this entailed 6 weeks of parenteral antibiotics, as such has always been concerned about her body weight. She
including the use of two antibiotics for Gram-positive organ- admits to use of diuretics in the past but denies recent use of
isms (19). This kind of prolonged, intensive regimen may in- diuretic, laxatives, or vomiting. Physical examination shows a
crease the chances of antibiotic-associated adverse effects and BP of 122/78 mmHg with no orthostatic changes. The remain-
perhaps bacterial resistance. Because catheter salvage with an- der of the examination is normal. Laboratory data are given in
tibiotics alone for 3 weeks will not be successful in the majority Table 1.
of cases, answers B and C (switch to oral antibiotics) are not the
best choices. Question 3
If a more definitive approach is to remove the dialysis cath- Which ONE of the following can BEST account for the clinical
eter, then this can be accomplished in two ways: The first findings in this patient (Figure 3)?
option is to remove the catheter, leave the patient without
A. Surreptitious vomiting
access for a day or two, and then re-insert a new catheter at the B. Surreptitious laxative abuse
original or at another site. This approach subjects the patients to C. Bartter syndrome
two procedures. Furthermore, in those with tenuous vascular D. Sjögren syndrome
access, there is a possibility that venous access will not be E. Surreptitious diuretic abuse
re-established at the re-insertion of the catheter. Patients may
also have to be hospitalized to expedite the two interventions. Discussion of Case 3 (Question 3)
Another approach is to remove the catheter but replace it over Surreptitious laxative abuse (B) is the best explanation for the
a guidewire at the same procedure (20). This both avoids an- findings in this case. The prevalence of this disorder has been
other intervention and preserves the same venous access. The estimated to be between 4 and 26% of patients with chronic
guidewire-exchange approach has been demonstrated to be as
efficacious as the two-step procedure (21–23) and does not
seem to leave the patient more vulnerable to metastatic infec-
Table 1. Laboratory data for case 3
tion (23). More recently, however, Mokrzycki et al. (24) sug-
gested that the guidewire exchange may not be as successful for Parameter Value
CRB that is caused by Staphylococcus aureus. In the case in
question, the causative organism was coagulase-negative Creatinine (mg/dl) 1.1
Staphylococcus, so answer A, guidewire exchange and 3 weeks BUN (mg/dl) 14
of parenteral antibiotics, is correct. Option D is partially correct, Serum electrolytes
insofar as the catheter could be removed and re-sited; however, (mEq/L)
this patient had limited vascular access, and re-siting a catheter Na⫹ 139
to the femoral vein would be less preferable and associated K⫹ 2.3
with a higher risk for complications. Cl⫺ 92
The final consideration is alluded to in option E, whether the HCO3 34
catheter could be kept in situ without a guidewire exchange by Urine electrolytes
using an antibiotic lock in the catheter lumen to kill the organ- (mEq/L):
isms in the bacterial biofilm. Very high concentrations of anti- Na⫹ 10
biotic are instilled into each port at the end of each HD session K⫹ 15
and left there until the beginning of the next session, when the
Cl⫺ 80
antibiotic solution is aspirated. This approach has salvaged
Urinalysis and urine Specific gravity 1.024, pH 6.8,
catheters in two thirds or more of CRB without the need for
culture ⫹ blood on dipstick, 20
guidewire exchange or catheter replacement (25,26); however,
RBC/hpf, 5 to 10 WBC/
once again, infection with Staphylococcus aureus seems to be
hpf; urine culture: no
resistant to this approach, with a low salvage rate and an
growth
unacceptable complication rate (26,27). In this case of CRB
Stone analysis Ammonium urate
caused by coagulase-negative Staphylococcus, however, option E BUN, blood urea nitrogen; hpf, high-power field; RBC, red
is an acceptable alternative to guidewire exchange. Because it is blood cells; WBC, white blood cells.
Clin J Am Soc Nephrol 5: 1141–1160, 2010 Nephrology Quiz and Questionnaire: 2009 1145

cretion exceeds the loss of actual or potential base in the stool so


that the net effect is development of metabolic alkalosis.
The large increase in urinary ammonium concentration can
predispose to the development of ammonium urate stones as
was seen in this patient. Dick et al. (31) described nine women
with laxative abuse complicated by ammonium urate calculi.
Measurement of urinary electrolytes in these patients showed
low values for Na⫹ and K⫹ but increased urinary Cl⫺ (negative
urinary anion gap), consistent with increased urinary ammo-
nium excretion. Intracellular acidosis as a cause of increased
renal ammoniagenesis was inferred by the finding of low uri-
nary citrate levels because intracellular acidosis stimulates
proximal citrate resorption. The authors postulated the low
urinary Na⫹ concentration that results from mild volume con-
traction allows for urate to bind to the abundant ammonium
ion and predispose to ammonium urate stone formation.
Figure 3. Answers from the membership, question 3. A similar mechanism may be responsible for endemic am-
monium urate bladder stones described in children from third-
world countries (32). These patients have an acidogenic diet
diarrhea (28). Although one typically associates diarrhea with that is low in phosphate as a result of the high cereal content
the development of a normal anion gap metabolic acidosis, and lack of animal protein intake. Increased ammonia is the
hypokalemic metabolic alkalosis can be the dominant acid-base primary buffer for H⫹ secretion, particularly because urinary
disturbance in these patients. phosphate excretion is low as a result of low animal protein
Acid-base disturbances that result from diarrhea can vary intake. The frequent occurrence of diarrhea predisposes these
depending on the site of origin of the diarrhea, whether the patients to low urinary volumes and low urine Na⫹ concentra-
diarrhea is secretory or osmotic, the duration of the disorder, tion. As in patients with laxative abuse, urate in the urine is free
and the degree of K⫹ deficiency (29). Under normal circum- to bind to the high concentrations of ammonium, leading to the
stances, stool electrolyte content is such that the sum of Na⫹ formation of ammonium urate calculi (Figure 4).
and K⫹ exceeds the concentration of Cl⫺. The resultant anion For years, the most commonly abused laxative was phenol-
gap can be accounted for by stool HCO3 and a variety of phthalein. Patients who used this laxative could be detected by
organic anions, many of which are derived from bacterial me- alkalinizing a sample of urine or stool and looking for the
tabolism of dietary carbohydrates. These organic anions are development of a strong violet color change. Several years ago,
absorbed and subsequently metabolized into HCO3 and there- the drug was reclassified by the Food and Drug Administration
fore represent a potential source of alkali. In the setting of acute so that it is no longer available as an over-the-counter agent.
diarrhea, particularly of small bowel origin, there is a signifi- The two most frequently abused laxatives today are bisaco-
cant loss of alkali from the body in the form of both HCO3 and dyl and anthraquinones (28). Bisacodyl is the active ingredient
potential HCO3 as a result of loss of organic acids. Despite the in Ex-Lax, Correctol, and Dulcolax. The most commonly used
development of systemic acidosis, the stool pH is typically anthraquinone is senna, which is the active ingredient in Seno-
acidic as a result of the high concentration of these acids. kot, Castoria, and Black Draft. Use of these laxatives can be
Surreptitious laxative abuse typically involves drugs that are detected in either stool or urine samples using thin-layer chro-
stimulants of the colon. Because colonic fluid has a high K⫹ matography.
content, patients who abuse these drugs tend to develop sig-
nificant total body K⫹ depletion (30). Hypokalemia can con-
tribute to the development of metabolic alkalosis in several
ways. First, the movement of K⫹ from the intracellular to
extracellular space in response to K⫹ depletion will be accom-
panied by movement of H⫹ into cells, resulting in intracellular
acidosis. The decrease in cell pH will serve as a stimulus for H⫹
secretion in the distal nephron of the kidney. Hypokalemia also
leads to increased expression of the H⫹-K⫹-ATPase in the distal
nephron. In the proximal tubule cell, acidification serves as a
major stimulus for increased synthesis of ammonia. The in-
crease in availability of ammonia to act as a urinary buffer
combined with an increase in H⫹ secretion by the distal
nephron leads to the generation of a metabolic alkalosis. The
urinary anion gap is negative in this situation as a result of the Figure 4. Proposed mechanism for ammonium urate renal
large increase in urinary NH4Cl excretion. Renal net acid ex- stone formation in a patient with long-term laxative abuse.
1146 Clinical Journal of the American Society of Nephrology Clin J Am Soc Nephrol 5: 1141–1160, 2010

Specimens that are submitted for laxative screening in the sion are normal. Right-sided pyelonephritis is diagnosed. The
United States are directed to a single reference laboratory. In patient is treated with intravenous ticarcillin/clavulanate, gen-
this regard, Shelton et al. (33) submitted stool and urine samples tamicin, and tetracycline, and the patient’s clinical condition
that were taken from normal volunteers in which diarrhea was gradually improves over the next several days. After a 2-week
induced by ingestion of either bisacodyl or senna to verify the course of parenteral antibiotics, she is discharged on no medi-
sensitivity and specificity of the assay used in this reference cations. One week after discharge, the patient presents with the
laboratory. Interestingly, the tests were reported as negative in complaint of weakness and paresthesias. Physical examination
all samples of urine and stool taken from the participants with shows downward beat nystagmus, and carpal pedal spasm is
senna-induced diarrhea. Testing of samples that were taken elicited. An electrocardiogram showed prominent U waves and
from participants with bisacodyl-induced diarrhea was more Q-T prolongation. Laboratory data are given in Table 2.
accurate but had sensitivity of only 73 and 91% and specificity
of 91 and 96% for the urine and stool samples, respectively. This Question 4
study suggests that testing for laxative abuse as currently prac- Which ONE of the following is the MOST likely cause of the
ticed can produce misleading results with a high frequency of electrolyte abnormalities in this patient (Figure 5)?
false-positive tests for bisacodyl and false-negative results for
A. Complication of ticarcillin/clavulanate
senna. It is important for clinicians to be aware of these limi-
B. Surreptitious diuretic use
tations in testing, because they evaluate patients with suspected
C. Tetracycline nephrotoxicity
laxative abuse.
D. Aminoglycoside nephrotoxicity
Vomiting (A) can certainly present as hypokalemic metabolic
E. Gitelman syndrome
alkalosis and should be suspected in a patient with possible
bulimia; however, the urine electrolytes would argue against
this diagnosis, and vomiting cannot account for the history of Discussion of Case 4 (Question 4)
ammonium urate nephrolithiasis. Vomiting leads to a contrac- Aminoglycoside nephrotoxicity (D) is the correct answer. This
tion of extracellular fluid volume, and loss of gastric acid gen- patient presents with the development of hypokalemic meta-
erates a metabolic alkalosis. While the patient is vomiting, the bolic alkalosis with normal BP after a recent hospitalization in
plasma HCO3 exceeds the tubular maximum for bicarbonate which she was treated with a variety of antibiotics for pyelo-
resorption in the proximal tubule, which leads to renal loss of nephritis. The electrolyte disturbance is accompanied by high
NaHCO3 (further exacerbating total body salt depletion) and normal values for plasma renin activity and aldosterone. Urinary
KHCO3 (leading to K⫹ depletion). The volume depletion leads calcium is increased, and the patient has hypomagnesemia.
to an increase in aldosterone secretion, which in the setting of Many of the features in this case are consistent with Bartter
high distal Na⫹ delivery accounts for the renal K⫹ wasting. syndrome, although the abnormalities in this patient are re-
Typical urinary electrolytes in active vomiting are a urine Cl⫺
⬍15 mEq/L, in the presence of a high urine Na⫹, a high urine
K⫹, and a urine pH of 7 to 8. Table 2. Laboratory data for case 4
Bartter syndrome (C) is also not correct. Although this dis-
order is characterized by hypokalemic metabolic alkalosis, Parameter Value
urine Na⫹, K⫹, and Cl⫺ all are increased. This pattern of urine
Creatinine (mg/dl) 0.9
electrolytes is also characteristic of active loop and thiazide
BUN (mg/dl) 14
diuretic use, making choice E incorrect. In addition, Bartter
Serum electrolytes (mEq/L)
syndrome and diuretic use would not account for the develop-
Na⫹ 139
ment of the renal stone disease described in this patient.
K⫹ 2.3
The two most common renal manifestations in Sjögren syn-
Cl⫺ 92
drome are nephrogenic diabetes insipidus and type 1 distal
HCO3 34
renal tubular acidosis (RTA). Although nephrolithiasis and
Urine electrolytes (mEq/L)
nephrocalcinosis are characteristic of patients with type 1 RTA,
Na⫹ 110
the stone composition is most commonly calcium phosphate.
K⫹ 35
The type of stone and the lack of a hyperchloremic metabolic
Cl⫺ 106
acidosis make a diagnosis of Sjögren syndrome untenable (D).
Serum magnesium (mg/dl) 0.9
Spot urine Ca2⫹/creatinine 0.53
Case 4: Biff F. Palmer (mg/mg)
A 35-year-old woman is admitted with left flank pain and fever
Other testing Plasma renin activity
for 2 days. Physical examination shows a toxic-appearing thin
2.4 ng/ml per h
woman. Vital signs are significant for a temperature of 38.5°C,
(0.8 to 2.5),
BP 100/68 mmHg, pulse 110, and respiratory rate of 24. Right-
aldosterone 235
sided costovetebral tenderness is present. An ultrasound of the
ng/dl (35 to 240)
abdomen shows normal-sized kidneys and no evidence of hy-
dronephrosis. Renal function and serum electrolytes on admis- BUN, blood urea nitrogen.
Clin J Am Soc Nephrol 5: 1141–1160, 2010 Nephrology Quiz and Questionnaire: 2009 1147

Figure 6. Gentamicin is a divalent cation and therefore has the


potential to bind to the basolateral Ca2⫹-sensing receptor in the
thick limb of Henle. This binding leads to inhibitory effects on
the apical transporters, thereby contributing to a Bartter-like
syndrome.

Figure 5. Answers from the membership, question 4.


One of the antibiotics used in the treatment of this patient
was ticarcillin. A complication of this drug when given to
cently acquired as evidenced by the normal serum electrolytes patients who are volume depleted is the development of hy-
on admission to the hospital. pokalemic metabolic alkalosis. The drug is excreted as a Na⫹
Bartter syndrome is a hereditary disorder characterized by salt and acts as a nonresorbable anion. In the setting of in-
renal salt wasting and hypokalemic metabolic alkalosis resem- creased aldosterone levels, the high distal Na⫹ delivery leads to
bling the features of chronic loop diuretic therapy. This disease increased renal K⫹ excretion. The increase in luminal electro-
results from gene defects that lead to decreased NaCl resorp- negativity in the distal nephron also stimulates H⫹ secretion,
tion in the thick ascending limb of Henle. thus leading to the generation of a metabolic alkalosis. Charac-
With relevance to this case, an acquired form of Bartter teristic urine electrolytes in this setting are increased urine Na⫹
syndrome can develop in association with the administration of and K⫹, a low urinary Cl⫺, and an acid urine pH. As long as
aminoglycoside antibiotics (34 –36). Chou et al. (36) described patients are kept euvolemic so that aldosterone levels are sup-
four patients who presented with marked paresthesia, muscle pressed, serum electrolytes remain normal when the drug is
weakness, and tetany after gentamicin therapy with total dos- given. The development of the electrolyte abnormalities well
ages ranging from 1.2 to 2.6 g. All four patients developed after being exposed to the drug as well as the urinary electro-
hypokalemia, metabolic alkalosis, hypomagnesemia with hy- lyte pattern in this patient exclude A.
permagnesuria, and hypercalciuria. These abnormalities even- As detailed in a recent review, tetracyclines may enter cells in
tually resolved over a period of 2 to 6 weeks. A similar Bartter- the proximal tubule through organic anion transporters from
like syndrome has been described in rats that were given either the apical or the basolateral side (38,39). Once inside the
gentamicin. In these animals, administration of the drug leads cell, these drugs can produce tubular injury by inhibiting ribo-
to increased urinary excretion of Na⫹, K⫹, Ca2⫹, and Mg2⫹ somal protein synthesis. Such injury may result in development
accompanied by decreased expression of the Na⫹-K⫹-2Cl⫺ co- of a proximal RTA usually associated with Fanconi syndrome.
transporter in the thick ascending limb (37). The biochemical The absence of hypokalemic normal gap metabolic acidosis and
findings in patients who have developed this form of nephro- other features of proximal tubular dysfunction make C incor-
toxicity resemble those seen in patients with gain-of-function rect.
mutations in the calcium-sensing receptor. Because gentamicin One last comment about this patient has to do with the
is a divalent cation, binding and activation of the calcium- neurologic manifestations noted on physical examination. One
sensing receptor may account for the development of this clin- of the clinical manifestation of hypomagnesemia is increased
ical syndrome (38) (Figure 6). neuromuscular irritability. In addition, hypomagnesemia is one
Gitelman syndrome is also characterized by hypokalemic of the few causes of downbeat nystagmus, as was seen in this
metabolic alkalosis; however, this diagnosis (E) can be excluded case (40,41).
by the normal electrolytes at the time of admission to the
hospital as well as the high urinary Ca2⫹ excretion. Hypocal- Case 5: Millie Samaniego
ciuria is the typical finding in patients with this disorder. A previously healthy 26-year-old white woman presented with
Surreptitious use of loop or thiazide diuretics can be a cause fever, vomiting, abdominal cramping, and diarrhea. Three
of hypokalemic metabolic alkalosis that is intermittent in nature weeks before this presentation, she noticed bruising over her
and cannot be totally excluded in this case; however, there was extremities followed by upper respiratory tract symptoms with
no history to suggest that this patient was prone to abuse of sore throat and rhinorrhea. Her husband and her 11-month-old
such drugs and therefore choice B would not be the best answer son had similar respiratory symptoms, and the husband re-
to explain the features in this case. ceived a diagnosis of culture-positive streptococcal pharyngitis.
1148 Clinical Journal of the American Society of Nephrology Clin J Am Soc Nephrol 5: 1141–1160, 2010

On presentation, the patient’s initial laboratory abnormalities D. Because her likelihood of recurrence is exceedingly high,
included a hemoglobin level of 6.7 g/dl, hematocrit of 18%, living-donor transplantation should be avoided and she
platelet count of 19,000/mm3, blood urea nitrogen level of 75 should be listed for a deceased-donor transplant.
mg/dl, and creatinine level of 4.6 mg/dl. Urinalysis showed 3⫹
protein and 3⫹ blood, and the urinary sediment was abnormal Discussion of Case 5 (Question 5A)
with ⬎100 red blood cells per high-power field and granular The most appropriate answer to this question is C: Additional
casts. A kidney biopsy showed diffuse capillary fibrin deposi- testing is necessary to provide an accurate estimate of the
tion and segmental staining for fibrin and IgM along the glo- likelihood of recurrence. HUS is characterized by microangio-
merular basement membrane (GBM), consistent with a diagno- pathic hemolytic anemia, thrombocytopenia, and acute kidney
sis of thrombotic microangiopathy (TMA). A disseminated injury. HUS is classified in two large categories: Diarrheal
intravascular coagulation workup was negative. Stool cultures (D⫹HUS) and nondiarrheal (D⫺HUS). Microvascular endothe-
for Escherichia coli O157:H7, Salmonella, Campylobacter, Shigella lial cell injury underlies both forms of this disease and leads to
species, ova, and parasites were negative. Other relevant tests the loss of the endothelium anticoagulant phenotype, enhanced
were a C3 level of 58 mg/dl (normal range 70 to 205) and C4 platelet consumption and leukocyte trafficking, complement
level of 20 (normal range 10 to 60); negative antinuclear anti- activation, and disruption of the fibrinolytic pathways (42).
body (ANA) and anti-streptolysin O titers; and negative anti- D⫹HUS more commonly affects children than adults, can
PR3, anti-myeloperoxidase, and anti-GBM antibody levels. Her present in an epidemic manner, and is usually associated with
ADAMST-13 activity was 97% (normal range ⱖ67) and inhibi- a prodrome of severe bloody diarrhea, hence its name. Endo-
tor ⬍0.4 units (normal range ⱕ0.4). thelial cell injury is caused by Shiga toxin–producing bacteria
She received a diagnosis of atypical hemolytic uremic syn- and can be sporadic, endemic, or epidemic. Escherichia coli
drome (aHUS) and was treated aggressively with 41 plasma O157:H7 strains are responsible for most cases of D⫹HUS. The
exchange sessions, four doses of rituximab, and a single dose of diarrhea in our patient is a red herring and likely represents a
vincristine without clinical response. She presents to your office nonspecific response to acute illness (42,43).
for preemptive transplant evaluation 7 months after the onset D⫺HUS, or aHUS, is more frequent in adults, can be sporadic
of disease and would like to know what the probability of or familial in presentation, and is unrelated to diarrhea. The
disease recurrence in the transplant is. past decade has seen the identification of mutations in the
genes for complement factor H (CFH), complement factor I
Question 5A (CFI), and membrane co-factor protein (MCP; or CD46) as
Given the clinical presentation of this patient, which ONE off the predisposing factors for the development of D⫺HUS. Approx-
following statements is MOST appropriate concerning the likeli- imately 30 to 50% of patients with D⫺HUS have genetic defect
hood of recurrence of disease in a renal allograft (Figure 7)? involving the complement regulatory proteins (CRPs). A grow-
A. The likelihood of recurrence is low, because the patient has ing number of mutations and polymorphisms that alone or in
nonfamilial HUS likely as a result of streptococcal infection. combination may lead to D⫺HUS are being identified. Some of
B. The likelihood of recurrence is high but will decrease with the less well-characterized genetic defects involve C3 and com-
time; therefore, delaying transplantation for 1 year is appro- plement factor B (44 – 48).
priate. CFH is a serum glycoprotein that is synthesized by the liver
C. Additional testing is necessary to provide an accurate esti- and regulates the function of the alternative pathway of com-
mate of the likelihood of recurrence. plement in fluid phase and on cellular surfaces. It binds to C3b,
accelerates the decay of the C3 convertase, and also acts as a
co-factor for CFI. The CFH gene is located on chromosome
1q32-q32.1 within a cluster of genes encoding regulatory com-
plement components (45). CFI is a serum regulatory serine
protease that is predominantly synthesized by the liver as a
single-chain precursor, the gene of which is located on chro-
mosome 4q25. MCP is a widely distributed C3b/C4b-binding
cell surface glycoprotein that serves as an inhibitor of comple-
ment activation via the classical and alternative pathways on
host cells. The genes for MCP are tightly clustered on chromo-
some 1 at q3.2 (46).
Deficiency or dysfunction of CRPs leads to complement-
mediated endothelial cell injury in the setting of unregulated
activity of the C3 and C5 convertases that leads to the produc-
tion of proinflammatory complement split products (C3a, iC3b,
C3dg, and C5a) and the membrane attack complex (C5b
through C9) (45).
Kidney transplantation in patients with HUS is associated
Figure 7. Answers from the membership, question 5A. with a variable rate of disease recurrence in the transplanted
Clin J Am Soc Nephrol 5: 1141–1160, 2010 Nephrology Quiz and Questionnaire: 2009 1149

graft, yet when recurrence occurs, it usually results in the loss crease with time; therefore, delaying transplantation for 1 year
of the transplant. Thus, an accurate diagnosis of the cause of is appropriate—is also incorrect. In patients with D⫹HUS or
HUS must be pursued before transplantation. Table 3 shows D⫺HUS as a result of MCP deficiency or drug-induced injury
the various rates of HUS recurrence on the basis of its different (e.g., ticlopidine, valacyclovir, clopidrogel), recurrence is negli-
causes (43,45,49,50). gible and kidney transplantation need not to be delayed
The diagnostic workup in this patient effectively ruled out (43,45,50).
D⫹HUS, because stool cultures for Shiga toxin–producing bac- In contrast, in patients with acquired (i.e., autoantibodies or
teria were negative, whereas negative lupus serologies and inhibitors) or hereditary deficiencies of CRPs, the risk for re-
ADAMST-13 inhibitor and normal ADAMST-13 activity ruled currence persists throughout the life of the individual unless
out sporadic forms of D-HUS such as systemic lupus erythem- the basic defect is corrected. Prevention of disease recurrence
atosus (SLE) and familial or sporadic thrombotic thrombocyto- and correction of CFH or CFI deficiency can be accomplished
penic purpura (51,52). Notably absent in the assessment of this with combined kidney-liver transplantation (46,53). Although
patient, however, was the evaluation for familial forms of the initial three reports that described this modality for CFH-
D-HUS or inhibitors of CRPs (CFH, CFI, MCH, and MCP au- associated HUS had poor outcome, more recent reports in
toantibodies) (44). which combined kidney-liver transplantation has been per-
Several findings in this patient suggest deficiency or dysfunc- formed successfully in combination with pretransplantation
tion of CRPs as the cause of D⫺HUS, including young age and plasma exchange have been described (53).
persistently low levels of C3 with normal C4 levels. In almost Current recommendations for combined kidney-liver trans-
all cases of D⫺HUS as a result of mutations in CFH and CFI, C4 plantation include CFH and CFI mutations, ⬍10% normal CFH
levels are normal and C3 levels are low, whereas in MCP levels in plasma, patients with identified mutations of CFH and
mutations, C4 and C3 levels are normal. Because of low sensi- CFI genes with recurrence after isolated kidney transplantation, or
tivity, the presence of normal C3 levels does not exclude mu- patients who have a family member with the same mutation and
tations of CRP genes (44,45). recurrence of HUS after isolated kidney transplantation (53).
Because patients with CRP deficiency or dysfunction lack a The correct answer is D: Because her likelihood of recurrence
classical clinical phenotype, this diagnosis is often missed is exceedingly high, living-donor transplantation should be
(46,49). Only in cases in which clinical suspicion prompts the avoided and she should be listed for a deceased-donor trans-
testing of serum activity and levels of CRPs as well as geno- plant. This choice is controversial. Similar to option B, the
typing can accurate diagnosis be reached. choice of pursuing or avoiding transplantation or the use of a
Choice A—the likelihood of recurrence is low because the deceased donor vìs a vìs a living donor depends on the cause of
patient has nonfamilial HUS likely as a result of streptococcal HUS and the likelihood of recurrence. In a meta-analysis of 10
infection—is the wrong answer. In patients with HUS, a history selected studies in 159 grafts of 127 patients with a well-docu-
of recurrent streptococcal infections or infections with other mented history of HUS in native kidneys and biopsy-proven
encapsulated microorganisms (e.g., Neisseria meningitidis, Hae- TMA in the transplanted kidney, living-donor transplantation
mophilus influenzae) suggests a familial etiology, especially CFH was associated with an increased risk for recurrence; however,
or CFI deficiency. In these patients, functional C3 deficiency as the risk for recurrence in living-related versus -unrelated kidney
a result of uncontrolled activation of the alternative pathway of transplants was not analyzed (54).
complement leads to deficient opsonization and phagocytosis Patients with D⫹HUS or D⫺HUS as a result of drug toxicity
of encapsulated organisms and hence infection (46). As shown should be offered living-donor transplantation. In patients with
in Table 3, CFH and CFI deficiencies are associated with high D⫺HUS as a result of CRP deficiencies or dysfunction, living-
recurrence of disease in kidney transplants and graft loss. unrelated donor transplantation should be discouraged given
Choice B—the likelihood of recurrence is high but will de- the high likelihood of recurrence and recurrence-related graft

Table 3. Rates of recurrence of HUS according to cause (46,47,50,53)


HUS in Native Kidneys Relative Risk of Recurrence in KTx (%)

Postdiarrheal HUS (D⫹HUS) Negligible


Nondiarrheal HUS (D⫺HUS) sporadic or familial forms
CFH mutation 80
CFI mutation 80 to 100
Idiopathic (unidentified genetic defect?) 30 to 50
MCP mutation 0 to 20
CFH autoantibodies 0
C3 and CFB deficiency ?
drugs, pregnancy, invasive S. pneumoniae infection Negligible
KTx, kidney transplantation.
1150 Clinical Journal of the American Society of Nephrology Clin J Am Soc Nephrol 5: 1141–1160, 2010

loss. Patients with MCP deficiency or anti-CFH autoantibodies, quantitative factor H level of 244.9 ␮g/ml (normal range 160 to
however, should be considered for living-unrelated kidney 412).
transplantation. MCP has a high level of expression in kidney
tissue, and transplantation with a living-donor kidney with Question 5B
normal (wild-type) MCP expression results in the cure of the Which ONE of the following choices would MOST likely con-
kidney disease. In patients with CFH autoantibodies, antibody- firm the correct diagnosis (Figure 9)?
depleting strategies in conjunction with isolated kidney trans-
A. Response to plasma exchange
plantation have been successful (46,50,53).
B. Donor-specific antibody (DSA) testing
Many centers do not recommend living-related transplanta-
C. Double-stranded DNA (dsDNA) antibodies
tion because of the risk for recurrence in the recipient and of de
D. Factor I and factor H genotyping
novo disease in the donor. In four reported cases, the donors
E. Discontinuation of calcineurin inhibitor (CNI)
went on to develop HUS within 1 year of donation. Genotyping
of the donor and recipient should be undertaken when living-
Discussion of Case 5 (Question 5B)
related donation is to be considered. This will not prevent the The correct answer is D: Factor I and factor H genotyping. The
risk for the donor in those with an unknown genetic basis current evidence suggests that all patients who have D⫺HUS
(46,50,53). Transplantation options for patients with aHUS are and are being considered for kidney transplantation should
shown in Figure 8. undergo screening for mutations in CRPs. An initial screening
One year after her first visit to your clinic, the patient received should test protein levels (either serum levels or surface expres-
a zero-antigen mismatch deceased-donor kidney transplant. The sion). This offers a rapid mechanism to identify the genes
immunosuppression consisted of alemtuzumab induction and involved (44,53).
maintenance therapy with prednisone, mycophenolate mofetil, In patients with normal levels of CRPs, genetic screening
and tacrolimus. She had excellent graft function initially with a should be performed on the basis of frequency of mutation
creatinine level of 1.1 mg/dl on discharge but on day 30 after (CFH approximately 30%; MCP approximately 10%; CFI 2 to
transplantation was admitted with a rise in serum creatinine to 5%). The exons in which CRP gene mutations cluster have been
1.9 mg/dl. A transplant biopsy showed TMA with many of the identified, and by screening these regions first, cost and detec-
19 glomeruli containing fibrin thrombi in a segmental distribu- tion time can be minimized (44,53). The patient presented here
tion. C4d staining was negative, and the tacrolimus trough was found to be heterozygous for a novel missense mutation,
level was 6 ng/ml. Further testing showed complement C3 of Y369S, in CFI (55).
70 (normal range 88 to 201) and C4 of 23 (normal range 16 to 47) Choice A—response to plasma exchange—is incorrect. Re-
and ADAMST-13 activity of 94% (normal ⬎67%) and with cent guidelines recommended the use of empiric plasma ther-
apy in all forms of D⫺HUS within 24 hours of diagnosis (53).
This is in contrast to previous guidelines in which the use of
plasma exchange was considered a grade C, level IV recom-
mendation given the lack of conclusive evidence of improve-
ment in outcomes (42).
Choice B—DSA testing—would not confirm the cause of this
patient’s disease. Although this patient may have been sensi-
tized against HLA antigens after receiving multiple sessions of

Figure 8. Therapeutic options for aHUS. IF, factor I; BF, factor B;


C3, complement protein 3; KTx, kidney transplantation; SKL,
simultaneous kidney-liver transplant; Eculizumab, humanized
mAb that functions as a complement inhibitor by blocking
cleavage of C5 into C5a and C5b and decreasing formation of
membrane attack complex. Figure 9. Answers from the membership, question 5B.
Clin J Am Soc Nephrol 5: 1141–1160, 2010 Nephrology Quiz and Questionnaire: 2009 1151

plasma exchange, C4 staining in the biopsy was negative, there-


fore making the diagnosis of antibody-mediated rejection
(AMR) less likely (56). It must be noted that fresh-frozen
plasma has been known to contain anti-HLA antibodies that
can lead to passive sensitization.
Choice C— dsDNA antibodies—is also incorrect. Although
active lupus can have a similar clinical presentation to D⫺HUS,
particularly in patients with antiphospholipid antibody syn-
drome, this patient does not have systemic evidence of lupus
activity (42). In lupus, both C3 and C4 levels are reduced, the
result of complement activation by autoantibodies via the clas-
sical pathway.
Choice E— discontinuation of CNI—would not lead to the
correct cause of this patient’s kidney disease. CNIs have been
identified as a cause of drug-induced HUS since the early
stages of their use in clinical transplantation. Recent studies
have now reported that some of the cases that were diagnosed Figure 10. Answers from the membership, question 6A.
as being CNI related may have actually been caused by AMR,
or ADAMST-13 inhibitors (57–59). The recently published
guidelines from the aHUS Consensus Study Group make no Discussion of Case 6 (Question 6A)
specific recommendations about use or avoidance of CNIs. The The correct answer is choice C: Antibody testing using single-
guidelines state that D⫺HUS is not considered per se a specific antigen bead flow cytometry should be performed emergently.
contraindication to treatment with CNIs and that initial immu- The patient has been sensitized to HLA antigens through mul-
nosuppression with mammalian target of rapamycin inhibitors tiple pregnancies. She is sensitized against class I HLA mole-
should not be encouraged (53). cules (i.e., PRA 20 to 79%) and highly sensitized against class II
HLA antigens (i.e., PRA ⱖ80%) and, accordingly, has a high
Case 6: Millie Samaniego immunologic risk for both T cell–mediated rejection and AMR.
A 55-year-old multiparous woman with type 2 diabetes re- Patients as the one discussed with high level of HLA sensitiza-
ceived a standard-criteria deceased-donor kidney transplant tion are likely to develop an anamnestic DSA response after
from a 20-year-old donor with a cold ischemic time of 6 hours. reintroduction of antigen in the form of a transplant. In such
Her class I and class II flow cytometry panel-reactive antibodies individuals, even if solid-phase assays such as flow-cytometry
(PRA) were 65 and 80%, respectively. A donor-specific flow cross-match are negative, low levels of DSA can be missed and
cross-match was negative. more sensitive solid-phase assays such as single-antigen bead
Her immunosuppressive regimen consisted of alemtuzumab Luminex flow cytometry are usually required (60,61).
induction and maintenance therapy with cyclosporine, myco- Choice A—DGF is likely given the prolonged cold ischemic
phenolate mofetil, and prednisone. Although urine production time—is incorrect. This patient received a deceased-donor
was noted after the vascular anastomosis was completed, the transplant from a young standard-criteria donor and a short
urine output decreased within the first 12 hours after transplan- cold ischemia time—all factors associated with a low risk for
tation and her blood urea nitrogen and creatinine failed to DGF (62,63). Although the risk for DGF increases in donors
decrease. A renal transplant ultrasound showed adequate who are older than 13 years, the donor age that is associated
blood flow in the renal transplant artery and vein and a resis- with a higher risk for DGF is ⱖ50 years. Although the risk for
tive index of 1.0. On day 2 after transplantation, the platelet DGF increases with cold ischemia time ⬎12 hours, cold isch-
count was noted to fall from 200,000 to 90,000/mm3, and her emia times ⬎36 (62) or ⬎40 hours (64) are associated with the
hemoglobin decreased from 11 to 8.5 g/dl. A repeat flow cy- highest risk.
tometry cross-match was negative. Choice B—CNI-related TMA is the most likely diagnosis—is
not a likely cause in this patient’s presentation. This patient has
clinical features suggestive of TMA: Acute anemia and throm-
Question 6A
Concerning the cause of this patient’s transplant dysfunction, which bocytopenia in the setting of transplant dysfunction and re-
ONE of the following statements is MOST correct (Figure 10)? duced urine output. Although CNIs are a recognized cause of
TMA, the onset of TMA is earlier than expected— usually be-
A. Delayed graft function (DGF) is likely given the prolonged yond day 4 after transplantation—although it can be highly
cold ischemic time. variable with a range of 4 to 2190 days after transplantation
B. CNI-related TMA is the most likely diagnosis. (65). Furthermore, severe forms of AMR can lead to glomerular
C. Antibody testing using single-antigen bead flow cytometry fibrinoid thrombosis (56), and some patients can have a TMA-
should be performed emergently. like syndrome.
D. Observant approach is adequate because this presentation is Choice D— observant approach is adequate because this pre-
consistent with alemtuzumab-related side effects. sentation is consistent with alemtuzumab-related side ef-
1152 Clinical Journal of the American Society of Nephrology Clin J Am Soc Nephrol 5: 1141–1160, 2010

fects—is a red herring. Alemtuzumab is an effective depleting terized by duplication of the GBMs, mesangial matrix expan-
humanized mAb directed against CD52—a surface marker of sion, and mesangial cell interposition (Figure 12). Originally
unknown function that is densely expressed in T and B lym- classified as a variant of chronic allograft nephropathy of un-
phocytes, monocytes/macrophages, and natural killer cells. As known cause, TG is now recognized with increased frequency
opposed to other depletional biologics, alemtuzumab is not in patients with a history of AMR and is also associated with
associated with anemia or thrombocytopenia, yet, in some pa- deposition of the complement split product C4d, suggesting
tients, the effect of alemtuzumab on CD52—a glycosylphos- that TG may be one manifestation of AMR (68,69). Gloor et al.
phatidylinositol-anchored protein—translates to other glyco- (70) reported a cumulative incidence of TG at 5 years of 20%
sylphosphatidylinositol proteins such as the CRPs CD55 and and a strong association with anti-HLA antibodies especially
CD59. The dysfunction of CRPs results in complement-depen- anti– class II, with the risk increasing when the antibodies were
dent intravascular hemolysis and thrombocytopenia, and case donor specific. Approximately 60% of patients with TG have
reports of HUS-like syndrome and TMA in alemtuzumab-treated proteinuria ⬎500 mg/d at the time of diagnosis, and, with
patients have been published (66,67). Regardless of its cause, TMA disease progression, patients can develop nephrosis, like the
in native or transplanted kidneys is an emergency, and observant patient described here. Per each 1-g increase in proteinuria, the
management is not indicated (42). hazard ratio of death-censored graft survival is 1.56 (95% con-
Single-antigen bead flow cytometry testing showed high lev- fidence interval 1.3 to 1.8; P ⬍ 0.0001) (68,70).
els of DSAs against class II antigen DR7, and a kidney trans- The remaining choices are incorrect. Although de novo mem-
plant biopsy confirmed the presence of DSA. The patient re- branous nephropathy, de novo FSGS, and recurrent diabetic
sponded favorably to aggressive therapy with plasmapheresis, nephropathy can result in nephrotic-range proteinuria, these
intravenous gammaglobulin, and intravenous steroids and was diseases usually present late in the natural history of kidney
discharged 2 weeks later with a serum creatinine level of 1.4 transplantation. De novo membranous nephropathy occurs ap-
mg/dl. At month 7 after transplantation, she presents to the proximately 49.27 ⫾ 32.71 months after transplantation, with a
clinic with a serum creatinine level of 2.5 mg/dl (compared cumulative incidence of 5.3% at 8 years after transplantation,
with 1.8 mg/dl the previous month) and a urine protein-to- and 62% of patients have nephrosis at the time of diagnosis. De
creatinine ratio of 3.5 (compared with 1.0 the previous month). novo FSGS was observed in 30% of kidney transplant recipients

Question 6B
Which ONE of the following choices is the MOST likely diag-
nosis for her proteinuria (Figure 11)?

A. De novo membranous nephropathy


B. Recurrent diabetic nephropathy
C. Transplant glomerulopathy (TG)
D. De novo FSGS
E. Acute T cell–mediated rejection

Discussion of Case 6 (Question 6B)


The correct choice is C: Transplant glomerulopathy. TG is a
condition that is associated with poor outcome and is charac-

Figure 12. Transplant glomerulopathy. (A) Global and severe


duplication of capillary walls with minimal, segmental mesan-
gial expansion (silver-stained section). (B) Mesangial prolifera-
tion, segmental endocapillary proliferation, cellular interposi-
tion, and global duplication of capillary walls (periodic acid-
Schiff–stained section). (C) Extensive remodeling of the GBM
with extensive reduplication of the GBM (electron microscopy).
(D) C4d-stained section in a biopsy with chronic active AMR
with positive peritubular capillaries and global glomerular cap-
Figure 11. Answers from the membership, question 6B. illary wall staining. Magnification, ⫻400.
Clin J Am Soc Nephrol 5: 1141–1160, 2010 Nephrology Quiz and Questionnaire: 2009 1153

during a mean follow-up of 57 months, and 24% of patients


exhibited nephrotic syndrome at the time of diagnosis (71).
Virtually 100% of patients with type 2 diabetes will develop
recurrence of diabetic nephropathy in the kidney transplant
with an average time to onset of de novo diabetic nephropathy
of 10 years (72). Finally, patients with acute T cell–mediated
rejection usually present with nonproteinuric or non-nephrotic
acute kidney transplant dysfunction. Of note, TG can be T cell
mediated, yet TG is a chronic, not an acute, histologic or clinical
cause of transplant dysfunction. That this patient had received
a diagnosis of AMR early in the course of her transplant makes
TG the most likely diagnosis, decreasing the likelihood of the
other options.

Case 7: Fernando C. Fervenza


In February 2009, a 64-year-old white woman was evaluated for
nephrotic syndrome. Serum creatinine level was 1.6 mg/dl and
proteinuria was 11 g/24 h. Tests for hepatitis B and C, cryo-
globulins, serum protein electrophoresis, ANCA, and comple-
ment levels were either negative or within normal range. ANA
test was 1:80. A renal biopsy showed a membranoproliferative
glomerulonephritis (MPGN) pattern of injury on light micros-
copy (Figure 13). On immunofluorescence, there was granular
capillary wall and mesangial staining for IgG (3⫹), IgM (2⫹),
C1q (1⫹), C3 (3⫹), ␬ (3⫹), and ␭ (trace) but no IgA (Figure 14).
Therapy with an angiotensin-converting enzyme inhibitor
(ACEI), aspirin, dipyridamole, and prednisone at 60 mg/d was
started with the dosage tapered during the subsequent 3
months. Despite the treatment, renal function continued to
deteriorate, and on June 21, serum creatinine had increased to
4.2 mg/dl. She was then referred for further evaluation and a

Figure 14. Immunofluorescence microscopy. (A and B) Granu-


lar staining for IgG (A) and ␬ (B) along the capillary walls. (C) ␭
Figure 13. MPGN. The glomerulus shows mesangial expansion light chain staining is negative.
with increase in mesangial matrix and cellularity, mostly as a
result of mononuclear cells, as well as endocapillary prolifera-
second opinion. On physical examination, BP was 139/82
tion. Capillary walls are thickened, and many loops show sub-
endothelial expansion and new basement membrane forma- mmHg, and, apart from peripheral edema, the physical exam-
tion, resulting in double contours and lobular accentuation of ination was unremarkable.
the glomerular tuft. Extracapillary proliferation, resulting in the Laboratory evaluation showed hemoglobin level of 11.7 g/dl,
formation of a small cellular crescent, is also present (silver leukocytes of 15.1 ⫻ 109/L, platelet count of 356 ⫻ 109/L,
stain). Magnification, ⫻400. albumin level of 2.3 g/dl, and serum creatinine level of 4.5
1154 Clinical Journal of the American Society of Nephrology Clin J Am Soc Nephrol 5: 1141–1160, 2010

mg/dl. Urinalysis showed ⬎100 red blood cells per high-power Table 4. Conditions associated with an MPGN pattern
field, ⬎25% dysmorphic, four to 10 white blood cells per high- of injury
power field, and a predictive 24-hour urine protein excretion of
17 g by urine protein-creatinine ratio. Serum protein electrophore- Chronic infections
sis found no monoclonal protein, but immunofixation revealed a viral: Hepatitis C, hepatitis B (rarely)
low-level monoclonal IgM ␭ plus a monoclonal IgG ␬. Free ␬ light bacterial: Endocarditis, infected ventriculo-atrial
chain was 4 mg/dl (0.33 to 1.95), free ␭ light chain was 5.65 mg/dl shunt, visceral abscesses, leprosy, meningococcal
(0.57 to 2.63), and ␬-␭ ratio was 0.7 (0.26 to 1.65). Serum C3 and C4 meningitis
complement levels were normal. An anti-dsDNA antibody was protozoa/other infections: Malaria, schistosomiasis,
negative. A repeat renal biopsy is performed. mycoplasma, leishmaniasis
Autoimmune diseases
Question 7 SLE
Which ONE of the following is the MOST likely diagnosis Sjögren syndrome
(Figure 15)? rheumatoid arthritis
C2 deficiency and SLE-like disease
A. Necrotizing and crescentic glomerulonephritis, pauci-im-
Inherited complement deficiencies (e.g., CFH, CFI, C3
mune
deficiency)
B. Cryoglobulinemic glomerulonephritis
Monoclonal gammopathies
C. MPGN secondary to a monoclonal gammopathy
Other neoplasias (e.g., lung cancer, lymphoma)
D. C1q nephropathy
Chronic and healed TMAs
E. Diffuse proliferative lupus nephritis (class IV)
healing phase of HUS/TTP
antiphospholipid (anticardiolipin) antibodies
Discussion of Case 7 (Question 7) syndrome
The correct answer is C. MPGN, or mesangiocapillary glomer-
radiation nephritis
ulonephritis, refers to a specific histologic pattern of glomerular
nephropathy associated with bone marrow
injury characterized by massive mesangial hypercellularity,
transplantation
mesangial matrix expansion, and thickening of the GBM with
drug-associated thrombotic angiopathies
the appearance of “double contours.” The duplicated or split
sickle cell anemia and polycythemia
basement membranes is the result of new basement membrane
dysfibrinogenemia and other prothrombotic states
formation caused by the interposition of the mesangial cell and
TG
mesangial matrix along the subendothelial side of the lamina
POEMS syndrome
densa. The proliferation in the mesangial region and mesangial
Idiopathic forms of MPGN
matrix expansion often has a lobular appearance. MPGN can be
MPGN type I
idiopathic or secondary to a variety of diseases (Table 4). Sys-
MPGN type II or dense-deposit disease
temic conditions that are associated with MPGN include auto-
MPGN type III/IV (Strife and Burkholder variants)
immune diseases (e.g., SLE) and chronic infections (e.g., hepa-
titis C). Less widely known, however, is the association of TTP, thrombotic thrombocytopenic purpura.
MPGN with monoclonal gammopathy. Monoclonal gammopa-
thy is defined as an excessive production of an Ig or its subunits
that arises from a clonal proliferation of plasma cells or B
lymphocytes. The majority of kidney diseases that are associ-
ated with monoclonal gammopathy are secondary to deposi-
tion of light chains (␬ or ␭) and not heavy chains or intact Igs
(73). These include myeloma kidney (cast nephropathy), Ig
light chain (AL) amyloidosis, and light chain deposition disease
(74). The spectrum of renal lesions that are associated with
monoclonal gammopathy is extensive and depends on the
physiochemical properties of the Ig produced (75). It should be
pointed out that whereas light chain deposition with an MPGN
pattern of injury is a widely known entity, MPGN secondary to
intact monoclonal Igs is less frequently recognized.
We recently analyzed renal biopsies of patients who received
a diagnosis of MPGN at the Mayo Clinic during a 6-year period
from 2001 through 2006 (76). Among the 65 patients with
hepatitis-negative MPGN, we identified 28 (43.1%) patients
who were positive for monoclonal/biclonal Igs. Immunofluo-
rescence microscopy showed granular immune deposits in the
Figure 15. Answers from the membership, question 7. mesangium and/or along the capillary walls, consisting of IgM
Clin J Am Soc Nephrol 5: 1141–1160, 2010 Nephrology Quiz and Questionnaire: 2009 1155

␬ (n ⫽ 11), IgG ␬ (n ⫽ 4), IgG only (n ⫽ 2), IgG ␭ (n ⫽ 1), IgM tion and capillary walls with double contours. In three glomer-
␭ (n ⫽ 1), and IgG/IgM ␭ (n ⫽ 1). These results correlated with uli, extracapillary proliferation resulted in the formation of
immunofixation results (in two cases, IgG was noted in the cellular crescents with fibrinoid necrosis. Immunofluorescent
mesangium and along capillary walls, but light chain restriction histology showed granular staining for IgG (2⫹), IgM (1 to 2⫹),
was not documented). In most biopsies, the deposits were more C1q (1 to 2⫹), C3 (3⫹), and ␬ light chain (3⫹) along the
prominent along the capillary walls than in the mesangium, capillary walls. ␭ light chain and IgA were negative. Electron
whereas in a few others the reverse was true. Three biopsies did microscopy showed no tubuloreticular structures and no mes-
not contain glomeruli or contained only globally sclerotic glo- angial deposits. These findings led us to make the diagnosis of
meruli, and in five biopsies, significant immune deposits were an MPGN secondary to a monoclonal gammopathy.
not noted; however, three of the five negative cases showed C3 A pauci-immune crescentic glomerulonephritis (A) is the
along the capillary walls. On electron microscopy examination, most common cause of rapidly progressive glomerulonephritis
there was thickening of the capillary walls with subendothelial in patients who are older than 60 years (80). In these cases,
deposits in all cases. Cellular interposition and new basement ANCA is present in ⬎90% of patients at initial presentation
membrane formation with double contours were also seen. The (81). Although nephrotic-range proteinuria can occur in the
deposits were granular, and substructures were typically ab- setting of ANCA-associated vasculitis, the absence of ANCA
sent. In four biopsies, scattered subepithelial deposits could be together with massive proteinuria makes this diagnosis un-
identified. The mesangium also contained electron-dense de- likely. Cryoglobulinemic glomerulonephritis (B) can present as
posits in 21 cases. Podocytes showed segmental effacement of part of a renal-dermatologic small-vessel vasculitis syndrome.
the foot processes, and many of the capillary loops showed Cryoglobulinemic glomerulonephritis is typically associated
leukocyte infiltration. Tubuloreticular structures were absent in with positive hepatitis C serology, positive cryoglobulins, and
the endothelial cells. low C4, all of which were absent in this case. C1q nephropathy
Sixteen of the 28 patients had a normal bone marrow biopsy (D) is a glomerulonephritis that is characterized by predomi-
and were classified as having a monoclonal gammopathy of nant mesangial C1q deposition but with other histologic fea-
unknown significance (MGUS). The diagnosis of MGUS re- tures resembling lupus nephritis (e.g., capillary wall thickening
quires a serum monoclonal paraprotein band of ⬍30 g/L; a recognizable as “wire loops,” a “full house” on immunofluo-
bone marrow biopsy that shows ⬍10% plasma cells; and ab- rescence microscopy, and electron-dense deposits in mesangial
sence of end organ damage such as lytic bone lesions, anemia, and subendothelial locations) (82). Clinical features of SLE are
hypercalcemia, and kidney failure. It is the most common absent at presentation, although some patients may develop
plasma cell disorder recognized and is a potential precursor for positive lupus serology and overt clinical lupus on follow-up.
multiple myeloma (77,78). Thus, the important finding of this In this case, renal biopsy showed no features suggestive of
study is the association of MPGN with MGUS. This study also lupus nephritis or C1q predominance, making this diagnosis
shows that in addition to MGUS, a membranoproliferative unlikely. In diffuse proliferative lupus nephritis (E) a positive
pattern of injury can be seen in the setting of conditions that are ANA is present in ⬎95% of patients; however, ANA lacks
associated with a monoclonal gammopathy, such as B cell specificity and can occur, usually in low titers, as a result of
lymphomas, chronic lymphocytic leukemia, and multiple my- cross-reactivity with other autoantibodies. Hematuria and pro-
eloma. In one case, serum protein electrophoresis studies were teinuria in the setting of lupus nephritis are usually associated
negative even though the renal biopsy suggested an MPGN with low complement levels, and anti-dsDNA antibodies are
secondary to monoclonal gammopathy. A few months after the strongly positive. This patient had normal complement levels
biopsy, serum immunofixation results turned positive for a and negative anti-dsDNA, making the diagnosis of lupus ne-
monoclonal gammopathy. Thus, MPGN may often be the first phritis less likely.
sign of the underlying lymphoplasmacytic disorder. Although There is no standard treatment for patients with MPGN
a direct relationship between the presence of a monoclonal associated with a monoclonal gammopathy. Conservative as
protein and the development of an MPGN remains to be well as immunosuppressive therapy with the use of corticoste-
proved, these observations point in that direction. roids (alone or in combination with an alkylating agent), tha-
Recently, Nasr et al. (79) described an entity of proliferative lidomide, bortezomib (Velcade), mycophenolate mofetil, cyclo-
glomerulonephritis associated with IgG deposition. That study, sporine, and rituximab have been used in a small number of
however, differs from our observations in that the deposits patients with variable outcomes (79). Prospective, controlled
were composed exclusively of monoclonal IgG and thus may studies of larger cohorts of patients with MPGN and monoclo-
belong to a subgroup of patients with monoclonal IgG. Bone nal gammopathy are needed to ascertain optimal therapy. At
marrow biopsy was performed in 22 of 37 patients, only one of present, treatment decisions will have to be made purely on the
whom showed multiple myeloma. Because a monoclonal gam- basis of clinical experience. In this case, the patient was treated
mopathy was identified in only 30% of the cases, the authors with two pulses of methylprednisolone (1 g each) followed by
did not associate the lesions with MGUS, lymphoproliferative prednisone 60 mg/d, tapered down to 40 mg/d at 4 weeks,
disease, or multiple myeloma. with subsequent tapering by 10 mg every 2 weeks until discon-
In this case, a repeat renal biopsy was performed at presen- tinuation. The patient also received concomitant treatment with
tation to our institution and showed glomeruli with mesangial cyclophosphamide (1 mg/kg per day, orally) for 3 months. Oral
expansion, increased cellularity, and endocapillary prolifera- sulfamethoxazole-trimethoprim was used for Pneumocystis ji-
1156 Clinical Journal of the American Society of Nephrology Clin J Am Soc Nephrol 5: 1141–1160, 2010

roveci pneumonia prophylaxis. When last seen in October 2009,


serum creatinine level was 1.8 mg/dl, proteinuria had come
down to 4.3 g/24 h, and urinary sediment was inactive.
In summary, many patients with an MPGN pattern of injury
on renal biopsy have an underlying monoclonal gammopathy,
and renal biopsies should be analyzed with anti–light chain
antibodies to detect a possible underlying monoclonal gam-
mopathy. Similarly, all patients with MPGN should undergo a
full workup for gammopathies, which should include serum
and urine immunofixation studies. When positive, a bone mar-
row biopsy should also be performed. As it stands, “idiopathic”
MPGN seems to be a vanishing disease and a possible under-
lying cause is likely to be found in the majority of cases of
MPGN.

Case 8: Fernando C. Fervenza


Figure 16. Answers from the membership, question 8.
A 60-year-old man with a history of hypertension and type 2
diabetes for 10 years was evaluated for sudden onset of massive
proteinuria (45 g/d). Renal biopsy showed 24 glomeruli, six of negative for either light chain. In this case, the renal biopsy
which were totally sclerotic. All of the glomeruli showed mes- finding are not compatible with the diagnosis of AL amyloid or
angial expansion with formation of homogeneous acellular pe- AA amyloid. If the amyloid protein cannot be typed into AL or
riodic acid-Schiff– and silver-negative nodules. Congo red AA subtypes, then it should raise suspicion for a hereditary
stains were positive and showed reddish brown material that amyloidosis and the diagnosis of a hereditary form should be
showed apple green birefringence under polarized light. Im- pursued by serum isoelectric focusing, DNA sequencing,
munofluorescence study showed glomerular staining for IgG and/or mass spectrometry of the amyloid protein (83) (Table 5).
(1⫹), IgA (trace to 1⫹), IgM (2 to 3⫹), C3 (2⫹), C1q (2⫹), fibrin Hereditary systemic amyloidoses are usually derived from
(2 to 3⫹), ␬ (trace to 1⫹), and ␭ (1⫹). Electron microscopy genetic variants of transthyretin (familial amyloid polyneurop-
showed mesangial expansion with amyloid fibrils. Staining for athy), apolipoprotein AI, apolipoprotein AII, lysozyme, cysta-
serum amyloid protein (SAP) was positive, but staining for tin C, gelsolin, TNF receptor, or fibrinogen A ␣-chain (AFib)
serum amyloid A (SAA) protein was negative. Serum immuno- (84). These diseases all are inherited in an autosomal dominant
fixation was negative, whereas urine immunoelectrophoresis manner with variable penetrance. Variations in the gene muta-
showed polyclonal ␬ and ␭ light chains. Bone marrow biopsy tions result in clinical amyloidosis syndromes that differ with
was negative for monoclonal plasma cells. respect to clinical presentation, age of onset, organ distribution,
rate of progression, and prognosis. Of these conditions, the
Question 8 most common is caused by mutations in the transthyretin gene.
Which ONE of the following is the MOST appropriate next step Patients usually have a family history of progressive peripheral
(Figure 16)? and autonomic neuropathy secondary to amyloid, although
involvement of the heart or kidneys is variable. Conversely, the
A. Proceed with high-dosage melphalan followed by autolo- kidney is usually involved in amyloidosis caused by mutations
gous stem cell transplantation (ASCT). in the genes that encode lysozyme, apolipoproteins AI and AII,
B. Begin treatment with vincristine, adriamycin, and dexa- or AFib (84); however, these autosomal dominant conditions
methasone for three cycles, then proceed with high-dosage are generally not considered in the differential diagnosis for
melphalan and ASCT. patients without a relevant family history. Lachmann et al. (85)
C. Conduct scintigraphy with labeled SAP component. reported on 350 patients who had systemic amyloidosis and for
D. Conduct genetic testing for hereditary amyloid variants. whom a diagnosis of AL amyloidosis had been made on the
E. Start oral colchicine and an ACEI. basis of clinical and laboratory findings and by the absence of
a family history. Testing these patients for amyloidogenic mu-
Discussion of Case 8 (Question 8) tations showed that mutations were present in 34 (9.7%) of the
Conduct genetic testing for hereditary amyloid variants (D) is 350 patients, most often in the genes that encode AFib (18
the correct answer. The diagnosis of amyloidosis is usually patients) and transthyretin (13 patients). In all 34 of these
based on positive Congo red and/or Thioflavin T stain. Routine patients, the diagnosis of hereditary amyloidosis was con-
subtyping of amyloidosis into AL amyloidosis (light chain– firmed by additional investigations. Of note, a low-grade
associated amyloid) and AA amyloidosis (secondary amyloid) monoclonal gammopathy (⬍0.2 g/dl) was detected in eight
is then done by immunochemistry staining for ␭ and ␬ light (24%) of the 34 patients, but in none of these patients were free
chains, SAP, and SAA protein. In AL amyloidosis, the amyloid light chains detected in the urine. Because MGUS can be found
stains for SAP and either ␬ or ␭ light chain. In case of AA in 3.2% of individuals who are 50 years of age and increases to
amyloidosis, the amyloid stains for SAP and SAA protein but is 5.3% among individuals who are aged ⱖ70 years (77), the
Clin J Am Soc Nephrol 5: 1141–1160, 2010 Nephrology Quiz and Questionnaire: 2009 1157

Table 5. Causes of systemic amyloidosis


Amyloid Protein Precursor Syndrome or Involved Tissue

ALa Ig light chain Primary; myeloma-associated


AHa Ig heavy chain Primary; myeloma-associated
A␤2Ma ␤2-microglobulin Hemodialysis-associated; joints
ATTR Transthyretin Familial; senile systemic
AA (Apo) serum AA Secondary, reactive
AApoAI Apolipoprotein AI Familial
AApoAIV Apolipoprotein AIV Sporadic, associated with aging
AGel Gelsolin Familial (Finnish)
ALys Lysozyme Familial
AFib Fibrinogen ␣-chain Familial
ACys Cystatin C Familial
ABri ABriPP Familial dementia, British
a
Can be localized/organ restricted.

detection of a monoclonal protein in a patient with the diagno- has been attempted as a way to prevent recurrence from he-
sis of amyloidosis may result in misleading the diagnosis as AL reditary AFib amyloidosis with some success (87,91). Liver
amyloidosis. In the report by Lachmann et al. (85), four of the 18 transplantation removes the source of the circulating fibrinogen
patients with AFib mutation had a monoclonal gammopathy; variant. Combined kidney-liver transplantation has also been
the presence of these monoclonal gammopathies reinforced the done in a case with no evidence of recurrence of AFib amyloid-
initial misdiagnosis of AL amyloidosis, and three of the four osis (92); however, it has been argued that because recurrent
patients received cytotoxic chemotherapy. Thus, it is important AFib amyloidosis develops slowly, liver transplantation should
that the diagnosis of hereditary amyloidosis be considered for all be considered only for patients or families in whom fast recur-
patients who receive a diagnosis of amyloidosis but for whom rence has resulted in accelerated loss in a first kidney transplant
biopsy stains are negative for the reactive systemic amyloid AA or when the involved mutation is known to result in fast
type and confirmation of the AL type cannot be obtained. recurrence in the graft (91).
In this case, subsequent DNA sequencing showed a mutation Light chain amyloidosis (AL), formerly known as primary
in valine 526 position of the AFib, leading to a diagnosis of amyloidosis, is the most common form of amyloidosis and can
hereditary AFib amyloidosis. Hereditary AFib amyloidosis has respond to therapy that is directed at the underlying plasma
been the subject of a number of publications (86 –90). Patients
cell dyscrasia (93–95). Thus, high-dosage melphalan followed
with AFib amyloidosis typically present in middle age with
by ASCT (A) would be appropriate if the patient had AL
proteinuria, hypertension, and progressive renal failure that
amyloidosis. In this case, the absence of a monoclonal protein
reaches ESRD within 4 to 8 years (87). A family history of renal
on serum and urine immunofixation studies together with a
disease is frequently absent. Renal biopsy often shows massive
bone marrow biopsy showing no monoclonal plasma cells rules
accumulation of amyloid in the glomeruli but little or no inter-
out the diagnosis of AL amyloidosis. The use of vincristine,
stitial or vascular amyloid. The most common amyloidogenic
adriamycin, and dexamethasone for induction before ASCT (B)
mutation in the fibrinogen gene is the single-base substitution
had been tested in 28 patients with AL amyloidosis (96). In this
of glutamic acid to valine in the codon at position 526 (as was
study, four patients died before ASCT (two from progression,
noted in our patient). Other mutations at codons 552, 540, 538,
and 525 have also been described (87). In patients with AFib two from treatment). No additional benefits were found in
amyloidosis, clinical manifestations are typically restricted to comparison with ASCT alone; therefore, induction chemother-
the kidney, with significant extrarenal disease being very rare. apy before ASCT is not recommended.
In the report by Gillmore et al. (87), median time from presen- Scintigraphy with labeled SAP component (C) has been used
tation to ESRD was 4.6 years, and the estimated median patient to quantify imaging of amyloid deposits in vivo but is nonspe-
survival was 15.2 years. Among 44 patients who reached ESRD, cific for any particular type of amyloidosis (97). Similarly, ther-
median survival was 9.3 years. Kidney transplantation alone apy with colchicine and an ACEI would be appropriate for
has been performed in a number of patients with AFib amy- patients with a diagnosis of familiar Mediterranean fever
loidosis, and in the series by Gillmore et al., 12 kidney trans- (FMF). FMF usually occurs during childhood, with 60% of
plants survived for a median of 6.0 years (range 0.0 to 12.2 patients showing first symptoms before the age of 10 years (98).
years), whereas seven grafts failed after a median follow-up FMF is clinically characterized by bouts of fever and serositis
from transplantation of 5.8 years; however, unless the supply of (peritonitis, synovitis, or pleuritis) that last 1 to 3 days (99). Skin
the abnormal amyloidogenic protein could be reduced, AFib involvement is also common, generally as an erysipela-like rash
amyloidosis will recur after a kidney transplant and may result that occurs most frequently on the shins and dorsum of the
in allograft loss (76,87). Combined kidney liver transplantation foot. AA amyloidosis can develop in a significant percentage of
1158 Clinical Journal of the American Society of Nephrology Clin J Am Soc Nephrol 5: 1141–1160, 2010

patients with FMF. None of these findings was present in this catheters eliminates paraspinal/vertebral infections in pa-
case, and stain for AA amyloid was negative. tients with catheter-associated bacteremia. Am J Nephrol 23:
202–207, 2003
19. Ashby D, Power A, Singh S, Choi P, Taube D, Duncan N,
Disclosures Cairns T: Bacteremia associated with tunnelled hemodial-
None. ysis catheters: Outcome after attempted salvage. Clin J Am
Soc Nephrol 4: 1601–1605, 2009
20. Carlisle E, Blake P, McCarthy F, Vas S, Uldall R: Septicemia
References in long-term jugular hemodialysis catheters: Eradicating
1. Friedman O, Jassal S, Bargman J: Acinetobacter peritoneal infection by changing the catheter over a guidewire. Int J
dialysis peritonitis: Description and relation to the SPICE Artif Organs 14: 150 –153, 1991
family of organisms. Perit Dial Int 28: 195–197, 2008 21. Shaffer D: Catheter-related sepsis complicating long-term,
2. Rodriguez-Bano J, Pascual A: Clinical significance of ex- tunnelled central venous dialysis catheters: Management
tended-spectrum beta-lactamases. Expert Rev Anti Infect by guidewire exchange. Am J Kidney Dis 25: 593–596, 1995
Ther 6: 671– 683, 2008 22. Robinson D, Suhocki P, Schwab S: Treatment of infected
3. Turner P: Extended-spectrum beta-lactamases. Clin Infect tunnelled venous access hemodialysis catheters with
Dis 41[Suppl 4]: S273–S275, 2005 guidewire exchange. Kidney Int 53: 1792–1794, 1998
4. Harada S, Ishii Y, Yamaguchi K: Extended-spectrum beta- 23. Tanriover B, Carlton D, Saddekni S, Hamrick K, Oser R,
lactamases: Implications for the clinical laboratory and Westfall A, Allon M: Bacteremia associated with tunnelled
therapy. Korean J Lab Med 28: 401– 412, 2008 dialysis catheters: Comparison of two treatment strategies.
5. Matuszkiewicz-Rowinska J: Update on fungal peritonitis and Kidney Int 57: 2151–2155, 2000
its treatment. Perit Dial Int 29[Suppl 2]: S161–S165, 2009 24. Mokrzycki M, Zhang M, Cohen H, Golestaneh L, Laut J,
6. Chou C, Kao M, Kuo H, Liu J, Liu Y, Huang C: Gram- Rosenberg S: Tunnelled haemodialysis catheter bacterae-
negative and polymicrobial peritonitis are associated with mia: Risk factors for bacteraemia recurrence, infectious
subsequent fungal peritonitis in CAPD patients. Perit Dial complications and mortality. Nephrol Dial Transplant 21:
Int 26: 607– 608, 2006 1024 –1031, 2006
7. Goldie S, Kiernan-Tridle L, Torres C, Gorban-Brennan N, 25. Vardhan A, Davies J, Daryanani I, Crowe A, McClelland P:
Dunne D, Kliger A, Finkelstein F: Fungal peritonitis in a Treatment of haemodialysis catheter-related infections.
large chronic peritoneal dialysis population: A report of 55 Nephrol Dial Transplant 17: 1149 –1150, 2002
episodes. Am J Kidney Dis 28: 86 –91, 1996 26. Poole C, Carlton D, Bimbo L, Allon M: Treatment of cath-
8. Zaruba K, Peters J, Jungbluth H: Successful prophylaxis for eter-related bacteraemia with an antibiotic lock protocol:
fungal peritonitis in patients on continuous ambulatory effect of bacterial pathogen. Nephrol Dial Transplant 19:
peritoneal dialysis: Six years’ experience. Am J Kidney Dis 1237–1244, 2004
17: 43– 46, 1991 27. Maya I, Carlton D, Estrada E, Allon M: Treatment of dial-
9. Wong P, Lo K, Tong G, Chan S, Lo M, Mak S, Wong A: ysis catheter-related Staphylococcus aureus bacteremia with
Prevention of fungal peritonitis with nystatin in patients an antibiotic lock: A quality improvement report. Am J
receiving CAPD. Perit Dial Int 27: 531–536, 2007 Kidney Dis 50: 289 –295, 2007
10. Piraino B, Bernardini J, Fried L, Johnston J: Pain due to 28. Duncan A: Screening for surreptitious laxative abuse. Ann
peritonitis. Perit Dial Int 19: 583–584, 1999 Clin Biochem 37: 1– 8, 2000
11. Divino Filho JC: Allergic reactions to icodextrin in patients 29. Perez GO, Oster JR, Rogers A: Acid-base disturbances in
with renal failure. Lancet 355: 1364 –1365, 2000 gastrointestinal disease. Dig Dis Sci 32: 1033–1043, 1987
12. Gokal R: Icodextrin-associated sterile peritonitis. Perit Dial 30. Oster JR, Materson BJ, Rogers AI: Laxative abuse syn-
Int 22: 445– 448, 2002 drome. Am J Gastroenterol 4: 451– 458, 1980
13. Elwell R, Manley H, Frye R, Bailie G: Pharmacokinetics of 31. Dick WH, Lingeman JE, Preminger GM, Smith LH, Wilson
intraperitoneal cefazolin and ceftazidime co-administered DM, Shirrell WL: Laxative abuse as a cause for ammonium
to CAPD patients. Int J Artif Organs 28: 808 – 816, 2005 urate renal calculi. J Urol 143: 244 –247, 1990
14. US Renal Data System: USRDS 2008 Annual Data Report, 32. Robertson WG: Renal stones in the tropics. Semin Nephrol
Bethesda, National Institute of Diabetes and Digestive and 23: 77– 87, 2003
Kidney Diseases, 2008 33. Shelton JH, Santa Ana CA, Thompson DR, Emmett M, Fordt-
15. Marr K, Sexton D, Conlon P, Corey G, Schwab S, Kirkland ran JS: Factitious diarrhea induced by stimulant laxatives:
K: Catheter-related bacteremia and outcome of attempted Accuracy of diagnosis by a clinical reference laboratory using
catheter salvage in patients undergoing hemodialysis. Ann thin layer chromatography. Clin Chem 53: 85–90, 2007
Intern Med 127: 275–280, 1999 34. Landau D, Kher KK: Gentamicin-induced Bartter-like syn-
16. Saad T: Bacteremia associated with tunnelled, cuffed he- drome. Pediatr Nephrol 11: 737–740, 1997
modialysis catheters. Am J Kidney Dis 34: 1114 –1124, 1999 35. Shiah CJ, Tsai DM, Liao ST, Siauw CP, Lee LS: Acute
17. Kovalik E, Raymond J, Albers F, Berkoben M, Butterly D, muscular paralysis in an adult with subclinical Bartter’s
Montella A, Conlon P: A clustering of epidural abscesses in syndrome associated with gentamicin administration. Am J
chronic hemodialysis patients: Risks of salvaging access Kidney Dis 24: 932–935, 1994
catheters in cases of infection. J Am Soc Nephrol 7: 2264 – 36. Chou CL, Yeong-Hwang C, Chau T, Lin S: Acquired Bar-
2267, 1996 tter-like syndrome associated with gentamicin administra-
18. Philipneri M, Al-Aly Z, Amin K, Gellens M, Bastani B: tion. Am J Med Sci 329: 144 –149, 2005
Routine replacement of tunneled, cuffed, hemodialysis 37. Sassen MC, Kim SW, Kwon TH, Knepper MA, Miller RT,
Clin J Am Soc Nephrol 5: 1141–1160, 2010 Nephrology Quiz and Questionnaire: 2009 1159

Frokiaer J, Nielsen S: Dysregulation of renal sodium trans- Fournier V, Bresson-Vautrin C, Chalopin JM: Recurrence of
porters in gentamicin-treated rats. Kidney Int 70: 1026 – hemolytic uremic syndrome in renal transplant recipients.
1037, 2006 Transplantation 65: 1405–1407, 1998
38. Zietse R, Zoutendijk R, Hoorn EJ: Fluid, electrolyte and 55. Chan MR, Thomas CP, Torrealba JR, Djamali A, Fernandez
acid-base disorders associated with antibiotic therapy. Nat LA, Nishimura C, Smith RJ, Samaniego MD: Recurrent
Rev Nephrol 5: 193–202, 2009 atypical hemolytic uremic syndrome associated with factor
39. Babu E, Takeda M, Nariwaka S, Kobayashi Y, Yamamoto I mutation in a living related renal transplant recipient.
T, Cha SH, Sekine T, Sakthisekaran D, Endou H: Human Am J Kidney Dis 53: 321–326, 2009
organic anion transporters mediate the transport of tetra- 56. Racusen LC, Colvin RB, Solez K, Mihatsch MJ, Halloran
cycline. Jpn J Pharmacol 88: 69 –76, 2002 PF, Campbell PM, Cecka MJ, Cosyns JP, Demetris AJ,
40. Saul RF, Selhorst JB: Downbeat nystagmus with magne- Fishbein MC, Fogo A, Furness P, Gibson IW, Glotz D,
sium depletion. Arch Neurol 38: 650 – 652, 1981 Hayry P, Hunsickern L, Kashgarian M, Kerman R, Magil
41. Du Pasquier R, Vingerhoets F, Safran AB, Landis T: Peri- AJ, Montgomery R, Morozumi K, Nickeleit V, Randhawa
odic downbeat nystagmus. Neurology 51: 1478 –1480, 1998 P, Regele H, Seron D, Seshan S, Sund S, Trpkov K: Anti-
42. Allford SL, Hunt BJ, Rose P, Machin SJ: Haemostasis and body-mediated rejection criteria: An addition to the
Thrombosis Task Force, British Committee for Standards Banff’97 classification of renal allograft rejection. Am J
in Haematology: Guidelines on the diagnosis and manage- Transplant 3: 708 –714, 2003
ment of the thrombotic microangiopathies haemolytic 57. Ulinski T, Charpentier A, Colombat M, Desconclois C,
anaemias. Br J Haematol 120: 556 –573, 2003 Mougenot B, Fremaux-Bacchi V, Suberbielle C, Deschênes
43. Ponticelli C, Banfi G: Thrombotic microangiopathy after G, Bensman A, Veyradier A: From humoral rejection to
kidney transplantation. Transpl Int 19: 789 –794, 2006 generalized thrombotic microangiopathy: Role of acquired
44. Kavanagh D, Richards A, Fremeaux-Bacchi V, Noris M, ADAMST-13 deficiency in a renal allograft recipient [erra-
Goodship T, Remuzzi G, Atkinson JP: Screening for com- tum appears in Am J Transplant 7: 730, 2007]. Am J Trans-
plement system abnormalities in patients with atypical plant 6: 3030 –3036, 2006
hemolytic uremic syndrome. Clin J Am Soc Nephrol 2: 591– 58. Pham PT, Danovitch GM, Wilkinson AH, Gritsch HA,
596, 2007 Pham PC, Eric TM, Kendrick E, Charles LR, Tsai HM:
45. Loirat C, Noris M, Fremaux-Bacchi V: Complement and Inhibitors of ADAMST13: A potential factor in the cause of
the atypical hemolytic uremic sydrome in children. Pediatr thrombotic microangiopathy in a renal allograft recipient.
Nephrol 23: 1957–1972, 2008 Transplantation 74: 1077–1080, 2002
46. Kavanagh D, Goodship T: Membrane cofactor protein and 59. Pham PT, Peng A, Wilkinson AH, Gritsch HA, Lassman C,
factor I: Mutations and transplantation. Semin Thromb Hae- Pham PC, Danovitch GM: Cyclosporine and tacrolimus-
most 32: 155–159, 2006 associated thrombotic microangiopathy. Am J Kidney Dis
47. Moore I, Strain L, Pappworth I, Kavanagh D, Barlow PN, 36: 844 – 850, 2000
Herbert AP, Schmidt CQ, Staniforth SJ, Holmes LV, Ward 60. Gebel HM, Bray RA, Nickerson P: Pre-transplant assess-
R, Morgan L, Goodship TH, Marchbank KJ: Association of ment of donor reactive, HLA-specific antibodies in renal
factor H autoantibodies with deletions of CFHR1, CFHR3, transplantation: contraindication vs risk. Am J Transplant 3:
CFHR4 and with mutations in CFH, CFI, CD46, and C3 in 1488 –1500, 2003
patients with atypical hemolytic uremic syndrome. Blood 61. Mihaylova A, Baltadjieva D, Boneva P, Ivanova M, Penkova K,
115: 379 –387, 2010 Marinova D, Mihailova S, Paskalev E, Simeonov P, Naumova E:
48. Le Quintrec M, Lionet A, Kamar N, Karras A, Barbier S, Clinical relevance of anti-HLA antibodies detected by
Buchler M, Fakhouri F, Provost F, Fridman WH, Thervet E, flow-cytometry bead-based assays: Single center experi-
Legendre C, Zuber J, Frémeaux-Bacchi V: Complement ence. Hum Immunol 67: 787–794, 2006
mutation-associated de novo thrombotic microangiopathy 62. Schnitzler MA, Hollenbeak CS, Woodward RS, Lowell JA,
following kidney transplantation. Am J Transplant 8: 1694 – Singer GG, Tesi RJ, Howard TK, Mohanakumar T, Brennan
1701, 2008 DC: The economic implications of HLA matching in cadav-
49. Ruggenenti P: Post-transplant hemolytic-uremic syndrome. Kid- eric renal transplantation. N Engl J Med 341: 1440 –1446, 1999
ney Int 62: 1093–1104, 2002 63. Irish W, Sherrill B, Brennan DC, Lowell J, Schnitzler M:
50. Loirat C, Fremeaux-Bacchi V: Hemolytic uremic syndrome Three-year post-transplant graft survival in renal-trans-
recurrence after renal transplantation. Pediatr Transplant 12: plant patients with graft function at 6 months receiving
619 – 629, 2008 tacrolimus or cyclosporine microemulsion within a triple-
51. Tsai H-M, Lian EC-Y: Antibodies to von Willebrand factor- drug regimen. Transplantation 76: 1686 –1690, 2003
cleaving protease in acute thrombotic thrombocytopenic 64. Irish WD, McCollum DA, Tesi RJ, Owen AB, Brennan DC,
purpura. N Engl J Med 339: 1585–1594, 1998 Bailly JE, Schnitzler MA: Nomogram for predicting the like-
52. Furlan M, Robles R, Galbusera M, Remuzzi G, Kyrle PA, lihood of delayed graft function in adult cadaveric renal
Brenner B, Krause M, Scharrer I, Aumann V, Mittler U, So- transplant recipients. J Am Soc Nephrol 14: 2967–2974, 2003
lenthaler M, Lämmle B: von Willebrand factor-cleaving pro- 65. Zarifian A, Meleg-Smith S, O’Donovan R, Tesi RJ, Batuman
tease in thrombotic thrombocytopenic purpura and the he- V: Cyclosporine-associated thrombotic microangiopathy in
molytic-uremic syndrome. N Engl J Med 339: 1578 –1584, 1998 renal allografts. Kidney Int 55: 2457–2466, 1999
53. Saland J, Ruggenenti P, Remuzzi G: Liver-kidney trans- 66. Taylor VC, Sims M, Brett S, Field M: Antibody selection
plantation to cure atypical hemolytic uremic syndrome. against CD52 produces paroxysmal nocturnal haemoglo-
J Am Soc Nephrol 20: 940 –949, 2009 binuria phenotype in human lymphocytes by a novel
54. Ducloux D, Rebibou JM, Semhoun-Ducloux S, Jamali M, mechanism. Biochem J 322: 919 –925, 1997
1160 Clinical Journal of the American Society of Nephrology Clin J Am Soc Nephrol 5: 1141–1160, 2010

67. Bonatti H, Brandacher G, Boesmueller C, Cont M, Hengster P, 83. El-Zoghby Z, Lager D, Gregoire J, Lewin M, Sethi S: Intra-
Rosenkranz AR, Krugmann J, Margreiter R: Hemolytic ure- tubular amyloidosis. Kidney Int 72: 1282–1288, 2007
mic syndrome following Campath-1H induction. Transpl Int 84. Picken MM: New insights into systemic amyloidosis: The
20: 386 –389, 2007 importance of diagnosis of specific type. Curr Opin Nephrol
68. Solez K, Colvin RB, Racusen LC, Sis B, Halloran PF, Birk Hypertens 16: 196 –203, 2007
PE, Campbell PM, Cascalho M, Collins AB, Demetris AJ, 85. Lachmann HJ, Booth DR, Booth SE, Bybee A, Gilbertson
Drachenberg CB, Gibson IW, Grimm PC, Haas M, Lerut E, JA, Gillmore JD, Pepys MB, Hawkins PN: Misdiagnosis of
Liapis H, Mannon RB, Marcus PB, Mengel M, Mihatsch MJ, hereditary amyloidosis as AL (primary) amyloidosis.
Nankivell BJ, Nickeleit V, Papadimitriou JC, Platt JL, N Engl J Med 346: 1786 –1791, 2002
Randhawa P, Roberts I, Salinas-Madriga L, Salomon DR, 86. Benson MD, Liepnieks J, Uemichi T, Wheeler G, Correa R:
Seron D, Sheaff M, Weening JJ: Banff ’05 meeting report: Hereditary renal amyloidosis associated with a mutant
Differential diagnosis of chronic allograft injury and elim- fibrinogen alpha-chain. Nat Genet 3: 252–255, 1993
ination of chronic allograft nephropathy (CAN). Am J 87. Gillmore JD, Lachmann HJ, Rowczenio D, Gilbertson JA,
Transplant 7: 518 –526, 2007 Zeng CH, Liu ZH, Li LS, Wechalekar A, Hawkins PN:
69. Sis B, Campbell PM, Mueller T, Hunter C, Cockfield SM, Diagnosis, pathogenesis, treatment, and prognosis of he-
Cruz J, Meng C, Wishart D, Solez K, Halloran PF: Trans- reditary fibrinogen A alpha-chain amyloidosis. J Am Soc
plant glomerulopathy, late-antibody-mediated rejection Nephrol 20: 444 – 451, 2009
and the ABCD tetrad in kidney allograft biopsies for cause. 88. Hamidi Asl L, Liepnieks JJ, Uemichi T, Rebibou JM,
Am J Transplant 7: 1743–1752, 2007 Justrabo E, Droz D, Mousson C, Chalopin JM, Benson MD,
70. Gloor JM, Sethi S, Stegall MD, Park WD, Moore SB, Delpech M, Grateau G: Renal amyloidosis with a frame
DeGoey S, Griffin MD, Larson TS, Cosio FG: Transplant shift mutation in fibrinogen alpha-chain gene producing a
glomerulopathy: Subclinical incidence and association novel amyloid protein. Blood 90: 4799 – 4805, 1997
with alloantibody. Am J Transplant 7: 2124 –2132, 2007 89. Lane SW, Goodman HJ, Francis L, Bofinger A, Mollee PN:
71. Ivanyi B: A primer on recurrent and de novo glomerulo- Hereditary fibrinogen A alpha-chain amyloidosis. Pathol-
nephritis in renal allografts. Nat Clin Pract Nephrol 4: 446 – ogy 38: 380 –382, 2006
457, 2008 90. Uemichi T, Liepnieks JJ, Benson MD: Hereditary renal
72. Wojciechowski D, Onozato ML, Gonin J: Rapid onset of
amyloidosis with a novel variant fibrinogen. J Clin Invest
diabetic nephropathy in three renal allografts despite nor-
93: 731–736, 1994
moglycemia. Clin Nephrol 71: 719 –724, 2009
91. Mousson C, Heyd B, Justrabo E, Rebibou JM, Tanter Y,
73. Sanders PW, Herrera GA: Monoclonal immunoglobulin
Miguet JP, Rifle G: Successful hepatorenal transplantation in
light chain-related renal diseases. Semin Nephrol 13: 324 –
hereditary amyloidosis caused by a frame-shift mutation in
341, 1993
fibrinogen A alpha-chain gene. Am J Transplant 6: 632–635, 2006
74. Leung N, Rajkumar SV: Renal manifestations of plasma
92. Gillmore JD, Booth DR, Rela M, Heaton ND, Rahman V,
cell disorders. Am J Kidney Dis 50: 155–165, 2007
Stangou AJ, Pepys MB, Hawkins PN: Curative hepatorenal
75. Audard V, Georges B, Vanhille P, Toly C, Deroure B,
transplantation in systemic amyloidosis caused by the
Fakhouri F, Cuvelier R, Belenfant X, Surin B, Aucouturier
Glu526Val fibrinogen alpha-chain variant in an English
P, Mougenot B, Ronco P: Renal lesions associated with
family. QJM 93: 269 –275, 2000
IgM-secreting monoclonal proliferations: Revisiting the
disease spectrum. Clin J Am Soc Nephrol 3: 1339 –1349, 2008 93. Gertz MA, Lacy MQ, Dispenzieri A, Hayman SR, Kumar S:
76. Sethi S, Fervenza FC, Miller D, Norby S, Leung N: Recur- Transplantation for amyloidosis. Curr Opin Oncol 19: 136 –
rence of amyloidosis in a kidney transplant. Am J Kidney 141, 2007
Dis February 21, 2010 [epub ahead of print] 94. Leung N, Dispenzieri A, Fervenza FC, Lacy MQ, Villicana
77. Kyle RA, Therneau TM, Rajkumar SV, Larson DR, Plevak R, Cavalcante JL, Gertz MA: Renal response after high-
MF, Offord JR, Dispenzieri A, Katzmann JA, Melton LJ 3rd: dose melphalan and stem cell transplantation is a favorable
Prevalence of monoclonal gammopathy of undetermined marker in patients with primary systemic amyloidosis.
significance. N Engl J Med 354: 1362–1369, 2006 Am J Kidney Dis 46: 270 –277, 2005
78. Kyle RA, Therneau TM, Rajkumar SV, Offord JR, Larson 95. Rajkumar SV, Gertz MA: Advances in the treatment of
DR, Plevak MF, Melton LJ 3rd: A long-term study of prog- amyloidosis. N Engl J Med 356: 2413–2415, 2007
nosis in monoclonal gammopathy of undetermined signif- 96. Perz JB, Schonland SO, Hundemer M, Kristen AV, Dengler
icance. N Engl J Med 346: 564 –569, 2002 TJ, Zeier M, Linke RP, Ho AD, Goldschmidt H: High-dose
79. Nasr SH, Satoskar A, Markowitz GS, Valeri AM, Appel GB, melphalan with autologous stem cell transplantation after
Stokes MB, Nadasdy T, D’Agati VD: Proliferative glomer- VAD induction chemotherapy for treatment of amyloid
ulonephritis with monoclonal IgG deposits. J Am Soc Neph- light chain amyloidosis: A single centre prospective phase
rol 20: 2055–2064, 2009 II study. Br J Haematol 127: 543–551, 2004
80. Jennette JC: Rapidly progressive crescentic glomerulone- 97. Hawkins PN, Lavender JP, Pepys MB: Evaluation of sys-
phritis. Kidney Int 63: 1164 –1177, 2003 temic amyloidosis by scintigraphy with 123I-labeled serum
81. Jennette JC, Falk RJ: New insight into the pathogenesis of amyloid P component. N Engl J Med 323: 508 –513, 1990
vasculitis associated with antineutrophil cytoplasmic auto- 98. Livneh A, Langevitz P, Zemer D, Padeh S, Migdal A, Sohar
antibodies. Curr Opin Rheumatol 20: 55– 60, 2008 E, Pras M: The changing face of familial Mediterranean
82. Sharman A, Furness P, Feehally J: Distinguishing C1q ne- fever. Semin Arthritis Rheum 26: 612– 627, 1996
phropathy from lupus nephritis. Nephrol Dial Transplant 19: 99. Drenth JP, van der Meer JW: Hereditary periodic fever.
1420 –1426, 2004 N Engl J Med 345: 1748 –1757, 2001

You might also like